boards

¡Supera tus tareas y exámenes ahora con Quizwiz!

Betwene the age of 70 -80 people lose what percent of their strength?

30%

According to the 2014 American Medical Association Code of Ethics Opinion, which statement is TRUE?

Individual gifts of minimal value from pharmaceutical representatives to physicians are permissible so long as the gifts are a benefit to patients.

Patients with primary brain tumors and intracranial metastases who undergo inpatient rehabilitation:

Individuals with brain tumors have significantly shorter lengths of stay. Their functional gains are similar to those of patients with acute stroke and traumatic brain injury.

Peak VO2 decreases with age, declining per decade by:

Peak VO2 declines by 8% to 10% per decade.

What is the most common fracture site for chronic SCI patients?

Femur fracture

Delivering a current through a monopolar needle inserted just lateral to the spinous process and overlying the vertebral lamina is known as?

Nerve root stimulation

describe a corellation between MRI findings and clinical outcome of spinal cord injury

no hemmorrhage on inital MRI correlates with a better prognosis and incomplete lesion

43. Chronic compartment syndrome can be confirmed with which test?

slit catheter

what type of cerebral palsy is most associated with a risk of epilepsy?

spastic quadriplegia

what stage of crps? severe burning at stage of injury, rapids nail and hair growth, and vasospasm

stage 1

These structures impinge what nerve? "Arcade of Struthers" bicipital aponeurosis Ligament of Struthers Arcade of Frohse

"Arcade of Struthers" is correct. The Arcade of Struthers is a piece of fascia connecting the brachialis to the triceps, and the ulnar nerve can become entrapped here. The bicipital aponeurosis and Ligament of Struthers cause median neuropathy at the elbow. The Arcade of Frohse causes posterior interosseous neuropathy.

65 year old female with left knee arthropalsty present with 3 days of left knee pain after falling onter her knee in the hospital parking lot. Although painful, she can extend her knee against ravity but not against resistance. Distally she is neurovascularly intact. A straight leg rais test is negative. Plain x ray imagesshowpatella alta with a speck of bone inferiorly. What is the next step in management?

- place in knee immobilizer for on operative management the patients ability to oppose gravity albeit weakly suggests an incomplete disruption of teh extensor mechanism. extensor mechanism and repair are unrelaible in the setting of prior arthroplasty. Furhtermore if teh extensor mecfhanism is intact as evidenced by the ability to perform a straight leg raise, non operative management is recommended, using a knee immobilizer to allow scar tissue to form before gradually increasing the patients range of motion.

Quick points about Leg calve perthes disease?

-more common in boys usually unilateral -pain may be referred to groin or knee -young children -does not always need surgery but definitely ortho referral promptly

What is the minimal ankle brachial index requirewd for succesful healing of a digital amputation?

0.45

A patient with a burn has undergone lower extremity grafting and immediate postoperative fitting with an edema control dressing. The earliest time for safe ambulation is how many days after the operation?

1

Anti double stranded DNA antibodies is associated iwth? Anti Ro and Anti la are primarily associated with? ANtinuclear antobodies are assoicated with?

1 = Lupus, SLE 2 = sjogrens 3 = not as specific for SLE

according to the Cantu return to play guidelines how long must a player sit out if she loses consciousness for more than 5 minutes after a concussion?

1 month

THe Miller Fischer variant of GLuillain Barre syndrome presents as a triad of which symptoms?

1 opthalmoplegia 2 areflexia 3 ataxia

physical findings in RA? try to name 9

1 ulnar deviation of digits 2 symmetric inflamation of small joints of hands 3 inflammation, most commonly of the MCP joints, wrists, PIP joints, knees, MTP joints, shoulders, ankles, cervical spoine, hip, elbow 4 TMJ 5tenosynovitis 6 carpal tunnel syndrome 7 swan neck deformityh 8 hammer toes 9 atrophy of teh muscles due to disease such as atrophy of the interosseous muscles of hands

Which of the following types of dermatomyositis is associated with cancer?

3

At which of the following ages in normal development does a child typically begin to walk?

12 months of age is typically when a child begins to walk (give or take a month).

patients who start glucocorticoids at andy dose who will cotninue for > or equal to 3 months shoudl be started on wht dose of Ca and Vit d?

1200- 1500 mg calcium and 800- 1000 units of Vit D per day

lupus patient whose symptoms are controlled on steroids presents with L 2 compression fracture. In regard to managing bone health, what should be done next?

1200- 1500 mg calcium and 800- 1000 units of Vit D per day and bisposphonate stopping steroids would be suboptimal as lupus symptoms would worsen with lower dose

intention or cerebellar tremor is how any Hz?

3 Hz mainly in the horizontal plane and is prominent with fine repetitive action of the extremities and is associated with other sign of cerebellar ataxia

Between ages 50 -70 people lose what percent of their strength?

15%

A typical botulinum toxin protocol for migraines consists of the following:

155 units divided into 31 sites This patient qualifies for botulinum toxin after failing abortive and prophylactic medications in addition to the frequency and duration of her migraines (>15 migraine days per month, lasting 4 or more hours). Typical migraine protocol for botulinum toxin consists of 155 units spread throughout 31 sites (5 units per site) in the head and neck. One of more injections into 7 muscle groups (corrugator, procerus, frontalis, temporalis, occipitalis, cervical paraspinal, trapezius) comprise the 31 total injections. Botulinum toxin injections are intended to provide relief for up to 3 months.

what puts a chi8ld at greater risk of spina bifida, mother with low folic acid intake or 2 siblings with spina bifida

2 siblings with spina bifida

how many days should elapse before using the same skin sight for a buprenorphine patch?

21 days

The decision to discontinue antiepileptic drug therapy in a patient with a history of epilepsy is appropriate when the patient has been seizure-free for at least how many months? A. 6 B. 12 C. 18 D. 24

24

The American Academy of Orthopedic Surgeons recommends weight loss to patients with symptomatic knee arthritis as a strategy to manage symptoms and improve function when the body mass index is calculated to be greater than or equal to ____ kg/m2:

25

What is the minmum recommended rating for pressure garments to reduce hypertrophic scarring in a patient with burn injury?

25 mm Hg

what percent of bone loss must you have to be able to see osteoporosis on X-ray?

25 to 30 percent

The decrease in cardiac mortality from participation in a program of cardiac rehabilitation is:

25%

rheumatoid degenerative changes of the subtalar joint occur in what percent of rheumatoid patients?

29-42%

. According to the American Spinal Injury Association (ASIA) International Standards for Neurological Classification of Spinal Cord Injury, testing of a key muscle should begin in which grade position?

3

What is the normal bladder capacity for a two-year old?

4 The bladder capacity in children is based on the Berger equation --age in years plus 2 -- equals bladder capacity in ounces up until around age 12 years when the bladder capacity equals an adult size. If the detrusor muscle is overactive and can overcome the pressure of the external sphincter, the result will likely be a small capacity bladder. If the external sphincter is open, there will also likely have a small capacity bladder unless there is have a large flaccid bladder.

what is the frequency of a parkinsonian tremor?

4-7 Hz

How many attacks per day would make youi think there is a secondary form of raynauds?

5 - 10 a day

center of gravity is where

5 cm anterior to S2, posterior to hip joint anterior to knee joint, posterior to patella anterior to ankle joint

essential tremor is how many Hz?

5- 8 Hz, more prominent with activity

Approximately 55% of adults (older than 20 years) with myelomeningocele report consistent bowel continence. Bowel continence has been positively associated with quality of life, employment, and educational attainment.

50 %

After major trauma, upper extremity fractures or surgery for both lower and upper extremity, what medication can be taken that may help prevent developement of CRPS?

50 day regimen of at least 500 mg daily vitamin C

How long is an abduction pillow utilized after a posterior approach hip replacement?

6 - 12 weeks

A 25-year-old runner reports having constant right shin pain of 3 weeks duration. She is currently running 40 miles/week and is training for her second marathon this year. X-ray of her right leg reveals no fracture line while her MRI shows bone marrow edema on T1 and T2 weighted images plus periosteal edema with no clear fracture line. Which of the following is the most appropriate recommendation for future activities?

6 -8 weeks non impact activities Runners with constant shin pain may have developed either medial tibial stress syndrome (MTSS or "shin splints") or tibial bone stress injury (TBSI or "stress fracture") due to a variety of factors related to overtraining , hyperpronation, low tibial bone density, etc. The Fredericson MRI criteria for MTSS and TBIS has been used to guide treatment. There are four grades (I-IV) with the patient in this question falling under Grade III. She would require 6-8 weeks of nonimpact activities.

At what age do children typically begin to manipulate objects across midline?

6 months

what CPP should be targeted to achieve the best outcome after TBI

60 mmHg

Transradial amputation is the most common level of upper limb amputation. Which level of amputation best allows for an externally powered prosthetic to be used?

60-70% residual limb length. At that length , good function and cosmesis is typically achievedm while also allowing for electronic componenet.

what prolonged supracapillary pressure does local soft tissue ischemia over a bony prominence occur? ? mmHg?

70 mmHg

What percent of rheumatoid arthritis patients are rheumatoid factor positive?

70-80% of rheumatoid arthritis patients are RF (+), which carries a poorer prognosis.

what percentage of max heart rate is commonly used target in an exercise tolerance test?

85

what stage of crps? muscle atrophy, soft tissue edema , thickening of skin and soft tissues and muscle waisting

stage 2

During repetitive nerve stimulation (RNS), what percentage decreases in the compound muscle action potential (CMAP) amplitude indicates that a neuromuscular junction (NMJ) disease is present?

A greater than 10% decrement in the CMAP amplitude during RNS is a positive finding for NMJ disease (myasthenia gravis, Lambert-Eaton Syndrome, botulism).

A 79 year-old female presents to you with the chief complaint of right foot pain. She denies numbness, tingling, or weakness. On exam you a note a few "hammer toes". How would you describe the position of a hammer toe? MTP: metatarsophalangeal joint. PIP: proximal interphalangeal joint. DIP: distal interphalangeal joint.

A Hammer toe is a toe demonstrating MTP extension, PIP flexion, and DIP extension, typically due to shoes being too small; treatment involves providing proper footwear with a toe box that is high and long enough to accommodate the toes.

A Jones fracture is a fracture across what?

A Jones fracture is a fracture across the base of the 5th metatarsal. Treatment is non-weight-bearing vs. surgery depending on type and severity.

A 17-year-old football player is the local high school's star running back. In the 2nd game of the season, he attempts to make a cut on a planted right foot. He feels a "pop" in his right knee and his leg gives out. He has immediate pain and swelling in his right knee and is unable to bear weight on that leg. MRI of the right knee the following day reveals a tear of his ACL (anterior cruciate ligament) and he undergoes surgical reconstruction. Which of the following orthoses is most appropriate post-operatively?

A Lenox-Hill derotation orthosis

A 20 year-old male presents to your clinic with the chief complaint of right foot pain. The pain occurred gradually without trauma. On exam there is slight swelling proximal to the 3rd toe. Foot x-rays demonstrate a 3rd metatarsal stress fracture. This is otherwise known by what name?

A metatarsal stress fracture is a March fracture. A Nutcracker fracture is a cuboid fracture and vice versa. A Jones fracture is a fracture across the base of the 5th metatarsal

A myopathic gait involves what?

A myopathic gait involves hyperlordosis, trendelenburg abnormality, and plantarflexed ankles no knee flexion!!!

Clinicians should approach patients with suspected limb-girdle muscular dystrophy (LGMD) by suggesting:

A clinical approach to guide genetic diagnosis based on clinical phenotype, inheritance pattern, and associated manifestations For patients with suspected muscular dystrophy, clinicians should use a clinical approach to guide genetic diagnosis based on clinical phenotype, inheritance pattern, and associated manifestations (Level B). Clinicians should refer newly diagnosed patients with an LGMD subtype and high risk of cardiac complications for cardiology evaluation even if they are asymptomatic from a cardiac standpoint (Level B). In patients with LGMD with a known high risk of respiratory failure, clinicians should obtain periodic pulmonary function testing (Level B). Clinicians should refer patients with muscular dystrophy to a clinic that has access to multiple specialties designed specifically to care for patients with neuromuscular disorders (Level B). Clinicians should not offer patients with LGMD gene therapy, myoblast transplantation, neutralizing antibody to myostatin, or growth hormone outside of a research study designed to determine efficacy and safety of the treatment (Level R).

A 30 year-old male is involved in a bicycle accident. He presents to the ED with right forearm pain. On exam he demonstrates intact sensation, but impaired strength secondary to severe pain. Trauma x-rays reveal a distal radius fracture in which the distal radius fragment is volarly displaced. This type of fracture is known by which of the following names?

A distal radius fracture is typically caused by trauma or fall on outstretched hand (FOOSH). Distal radius fractures happen in two ways: the distal radius fragment is dorsally displaced (Colles fracture), or volarly displaced (Smith fracture). Both require orthopedic surgery consultation.

You are evaluating a 16 year-old male in your outpatient clinic. He is noted to have scoliosis. He relies on a wheelchair for mobility. On examination he has impaired coordination and dysmetria. What is his most concerning issue in the future?

A patient with this constellation of symptoms most likely has Friedreich Ataxia. These patients usually die from heart disease (HOCM) and thus, they require careful cardiac workup.

A 22 year-old male sustains a lateral ankle sprain. Anterior drawer of the ankle, and talar tilt tests are positive. You grab your ultrasound machine because you are concerned about a secondary injury to which of the following structures most likely?

A possible complication of lateral ankle sprains is a fibularis longus or brevis tendon injury. This is due to the fact that these tendons wrap around the lateral malleolus; thus, severe inversion forces can cause abrupt stretching and possible tearing of these tendons. Diagnostic ultrasound scan would be useful as an in-office, quick evaluation of the tendon anatomy.

an example of a semantic paraphasia?

A semantic paraphasia is the act of substituting a word with another from the same semantic class (i.e. orange instead of apple, or above example pencil instead of pen). A neologism is the creation of a new word that has meaning only to the speaker. Echolalia is when someone repeats words/sounds made by someone else. Using words with similar sounds, such as "dock" instead of sock", is an example of phonemic paraphasia.

During electrodiagnostic studies, the A-wave can be studied. Describe the characteristics of an A wave and what it means?

A-waves (also called the axon reflex - not a true reflex) are a very predictable, stable waveform that shows up somewhere between the F-wave and the direct motor response when recording F-waves from a muscle. It is the exact same waveform with every stimulation (same latency and amplitude). If present, A-waves indicate that there has been previous reinnervation involving the nerve to the muscle being studied.

The most common structure involved in a segond fracture is what? Followed by what?

ACL followed by meniscal

what difference in amplitude would be seen between AE and BE in cubital tunnel syndrome

AE BE difference greater than 15 m/s or 23 %

Abnormal jitter seen during single fiber electromyography would indicate which of the following? ALS guillain-Barre Syndrome pseudobulbar syndrome CHarcot MArie Tooth

ALS jitter usually seen during the first month in disorders of teh neuromuscluar junction, amyotrophic lateral sclerosis, myopahties and axonal neuropathies

wHAT IS THE LIKELY REASON THAT THE sach FOOT HS BEEN DISCONTINUED?

AN overloading on the non amputated foot

Neuropathy is associated with cryoglobulinemia in the setting of?

Hep C cryoglobulinemia type 1 is usually associated with malignancy and type 2 and 3 are in chronic inflammatory condition and Hep C

Which corticosteroid has the longest duration of anti-inflammatory activity?

Betamethasone

Regarding pediatric prosthetics, approximately how often should a left transradial prosthesis be prescribed between ages 14 and 20?

Between ages 0-5 the prosthesis should be replaced approximately annually. Between ages 5-12 the prosthesis should be replaced approximately every 18 months. Between ages 12-21 the prosthesis should be replaced approximately every 2 years.

Lowering the high frequency filter during NCS will cause which of the following changes?

Lowering the high frequency filter during NCS will cause prolonged peak and onset latencies.

Which of the following muscles receives its innervation from all roots (C5-T1) of the brachial plexus? Flexor digitorum profundus Latissimus dorsi Pectoralis major

B Pectoralis major The pectoralis major is innervated by C5-T1 roots of the brachial plexus. Triceps and latissimus dorsi are innervated by C6, C7, C8 roots. FDP is innervated by C7, C8, T1 roots.

Lubiprostone is used to treat constipation through the following mechanism:

Activating type 2 chloride channels to promote gastrointestinal motility Lubiprostone (Amitiza) is a newer medication that treats chronic constipation by increasing intestinal fluid secretion and promoting gastrointestinal motility through the activation of type 2 chloride channels. This improves transit and passage of stool.Peripheral mu-receptor antagonists (a) like methylnaltrexone are used to treat constipation related to opiate use. This action improves constipation without reversing analgesia. Docusate (c) and other surfactant laxatives lower the surface tension of stool, allowing water and lipids to penetrate and soften the stool. Bisacodyl (d) and other stimulant laxatives directly stimulate nerves in the colon to increase peristalsis

What medical condition would demonstrate the feature shown in this motor nerve conduction study?

Acute inflammatory demyelinating polyradiculoneuropathy (AIDP) The tracings above demonstrate temporal dispersion, which is pathognomonic for an acquired demyelinating disease, and is among the diagnostic criteria for acute inflammatory demyelinating polyradiculoneuropathy (AIDP), chronic inflammatory demyelinating polyradiculoneuropathy (CIPD), multifocal motor neuropathy (MMN), multifocal acquired demyelinating sensory and motor (MADSAM) neuropathy or Lewis-Sumner syndrome, and distal acquired demyelinating symmetric (DADS). Demyelination is not a feature of acute motor axonal neuropathy (AMAN), therefore temporal dispersion is not pathognomonic. Diabetic polyneuropathy and mononeuropathy multiplex are axon loss neuropathies.

Once a patient has reached age 50, the conduction velocity of their nerves will decrease by approximately how many meters per second for each subsequent decade?

After age 50, conduction velocity normally decreases by about 2 m/s per decade.

bad side effect of carbamazepine

Agranulocytosis

All of the following muscles are knee flexors except which of the following? Sartorius Semitendinosus Biceps femoris Tensor fascia lata

All of the following muscles cross the knee joint posteriorly, rendering them knee flexors, except for tensor fascia lata, which does not directly cross the knee joint, but does control the IT band distally and is a hip abductor and internal rotator. The IT band itself distally crosses the knee anteriorly and attaches to Gerdy's tubercle of the tibia, and this does not provide a knee flexion force.

WHat is the most common cause of dementia in the US?

Alzheimer dementia

What is the preferred front line pharmacologic therapy for fibromyalgia?

Amytriptiline, low dose TCA

Which finding correlates with a high risk of ischemic ulceration?

Ankle/brachial ratio of 0.40

Radiographic findings associated with ankylosing spondylitis include symmetric SI joint narrowing, osteopenia, and a "Bamboo spine." A "Bamboo spine" refers to ossification of which structure?

Annulus fibrosis

Which best describes anosognosia? ​

Anosognosia is defined as a disturbance of a person's ability to recognize disability or illness in oneself. It is thought to be related to hemineglect, a tendency to ignore the contralateral half of one's body or of external space, that follows damage to the parietal lobe (or rarely, the diencephalon), right more often than left. Patients may not recognize the hemiplegia (anosognosia), their arm (asomatognosia), or any external object to the left of their own midline. These phenomena may occur without sensory defects in patients who are otherwise neurocognitively intact.

The strongest restraint to valgus stress in the elbow is the:

Anterior bundle of the medial collateral ligament The medial (ulnar) collateral ligament (also known as the Tommy John ligament due to rupture in throwing athletes) is the primary restraint due to valgus force. Specifically, the anterior oblique band is loose in flexion and taut in extension.

Shoulder dislocations most commonly occur in which of the following directions?

Anterior shoulder dislocations are the most common, followed by posterior dislocations.

bilateral PCA stroke leads to what syndrome with what symptoms?

Anton syndrome. cortical blindness. patients may deny cortical blindness

Inheritance pattern for myotonic dystrophy type 1 and myotonic muscular dystrophy type 2?

Autosomal dominant

What is the most common inheritance patter of hereditary neuropathy with liability to pressure palsy?

Autosomal dominant associated with PMP22

Prednisone functions by directly inhibiting the production of which of the following compounds?

Arachidonic acid Corticosteroids, such as prednisone, function by inhibiting the enzyme phospholipase A2 (PLA-2), which normally produces arachidonic acid. Without arachidonic acid, prostaglandins are also no longer produced. However, the direct effect of prednisone is to inhibit arachidonic acid production, which indirectly leads to decreased prostaglandins. NSAIDs directly inhibit the COX-1 and COX-2 enzymes, the effect of which directly inhibits the production of prostaglandins.

Which of the following is a feature of a complex repetitive discharge?

Arises from ephaptic activation of muscle fibers

A 22 year-old male presents to your clinic with the chief complaint of right foot and ankle pain with intermittent swelling. The pain is worse with athletic activity, and improved with rest. There is tingling in the dorsum of the foot. On exam, the dorsalis pedis pulse diminishes when the foot is placed into plantarflexion. What is the most appropriate diagnostic test to perform?

Arteriogram This patient is a classic presentation of popliteal artery entrapment syndrome (PAES), which involves the popliteal artery in the knee becoming compressed in the popliteal fossa due to an abnormal arterial course or knee muscles impinging upon it abnormally. It presents with lower limb swelling and pain with paresthesias due to nerve compression. The classic physical exam maneuver is to demonstrate decreased dorsalis pedis pulse when the ankle is plantarflexed (thus putting the gastrocnemius in a position to pinch the popliteal artery). Workup involves arteriogram to demonstrate abnormal arterial course or flow.

The most common etiology for ischemic stroke in children is:

Arteriopathy Arteriopathy is the most common cause of arterial ischemic stroke in children. Cardiac disease (congenital or acquired) and prothrombotic conditions (such as Factor V Leiden) are also causes of ischemic stroke in children. Arteriovenous malformations are a common cause of hemorrhagic (not ischemic) stroke in children

What is the consequence of placing the electrodes 2 cm away from active elctrode rather than 4 cm during SNAP?

Artificially low amplitude

A 23 year-old patient is involved in a motor vehicle accident and suffers a unilateral cervical facet joint dislocation ("jumped facet"). This most likely occurred at what level of the spine?

As a general rule it is always useful to remember that most pathologies in the cervical spinal cord occur at the C5 or C5-C6 level.

An 80-year-old woman with a history of dementia diagnosed three years ago is brought to your clinic by her family. The patient had been gradually deteriorating but has been coping at home with family support. She is confused at baseline. Two days ago, she became incontinent of urine (unusual for her), more confused with occasional return to baseline cognition, disorganized and inattentive in her conversation with family. The family has noted increased lethargy. Balance has remained unchanged from baseline. Her change in symptoms is most consistent with:

As this patient has dementia, she is likely to be at a particularly high risk of delirium. Worsening dementia is likely, due to the long history and gradual deterioration, but not due to the increased confusion over the last two days. Increased sleepiness over two days (recent) is not conclusive of depression.

At rest, inhalation is a ______ process while exhalation is a ______ process.

At rest, inhalation is an active process initiated by diaphragm contraction, while expiration is (at rest) a passive process.

Which of the following is good practice to prevent electrical injury during electrodiagnostic studies?

Avoid extension cords Leakage current increases with the length of the power cord, therefore extension cords should be avoided. A is incorrect because ground electrodes should always be used to dissipate leakage current, therefore nerve conduction studies are performed with a reference electrode, and active electrode, and a ground electrode. The machine should be turned on before attaching electrodes to the patient and after disconnecting them. Using a stimulus pulse duration greater than 0.2ms in someone with an implanted pacemaker or cardioverter increases the risk that the stimulus may be misinterpreted as a QRS complex.

The supraspinatus and deltoid contribute how many degrees to the function of shoulder abduction, respectively?

B 15, 165

25 year old female with patellofemoral pain syndrome. A) MRI confirms the diagnosis B) overuse is the most common mechanism of injury C) plain radiograph are usually diagnostic

B) overuse is the most common mechanism of injury common overuse injury in athletes

Diabetic patient has diabetic foot ulcers ( severe) . Which of the following is the most important component of the management of these ulcers?

REMOVAL OF ALL NECROTIC TISSUE AND DEBRIS wound irrigation would follow debridement.

65 year old lady with left hip pain after ground level fall presents and is now unable to bear weight without having significant pain. X rays of hip are normal. What is the best next step in management?

Hip MRI if there is clinical suspicion of a hip fracture with noraml X ray, T 1 weighted MRI is the best sequence of images for identifying fracture

Lateral epicondylitis with pain that radiates down the forearm and is associated with overuse.. initial treatment is?

RICE and strength training

what med has been shown to prolong survival in ALS patients

RIluzole

Osteoporosis is defined as which of the following on a DXA scan?

Osteoporosis is defined as under 2.5 standard deviations away from average peak bone mass. Normal range on DXA scan is a T score between -1 and 1. Osteopenia is defined as T score between -2.5 and -1. Osteoporosis is a T score less than -2.5.

if patient has hamate fracture, treat by?

Refer to ortho

The American Geriatrics Society's goal of diabetic control in an otherwise healthy elderly individual includes:

Blood pressure below 140/90 American Geriatric Society guidelines place greater emphasis on reduction of cardiovascular complications and minimizing the risk of hypoglycemia. In an otherwise healthy elderly individual, fasting plasma glucose below 130 mg/dL and a hemoglobin A1c of less than 7.5% is sought.

how is brown sequard syndrome plus different from brown sequard?

BSPS is much more common than brown sequard syndrome. patients with BSS have greatest prognosis for ambulation and functional outcome. They also have a favorable recovery of bowel and bladder function. Motor recovery in BSPS usually takes place in the ipsilateral proximal extensors and then the distal flexors.

Orthostasis in the elderly can be exacerbated by decreased:

Baroreceptor response As many as 20% of persons over the age of 65 years and 30% of those over age 75 have orthostatic hypotension. Orthostasis is often exacerbated because of an increase in arterial stiffness, increased peripheral resistance, decreased baroreceptor response, and low plasma renin activity.

Which therapy approach is based on the concept that pathological movement patterns must not be used for training because continuous use of the pathologic pathways may make it too readily available to use at the expense of the normal pathways.

Bobath

Along with nonuniform joint space narrowing, which of the following is a characteristic of osteoarthritis ? Bony sclerosis Pannus formation Atlantoaxial subluxation juxta articular osteopenia

Bony sclerosis Nonuniform joint space loss in association with bony sclerosis and marginal osteophytes is the characteristic change in osteoarthritis. Pannus is characteristic of rheumatoid arthritis; atlantoaxial subluxation is noted in rheumatoid arthritis, and therefore radiographic studies should be obtained in patients with cervicospinal complaints with RA; and juxta-articular osteopenia is commonly seen in RA.

signs of intracranial pressure for BP and HR?

Brady, hypertension

You are reviewing the medical chart for your next clinic patient. The patient is a 35-year-old female with a history of R breast cancer which was treated with partial mastectomy. You are consulted for lymphedema in the RUE since the surgery. The oncologist note states that she has grade I lymphedema. What does grade I lymphedema represent clinically?

Breast cancer (and its treatment) is the most common cause of upper extremity lymphedema. In grade I lymphedema, the edema is reversible with limb elevation. In grade 2 lymphedema, the edema is reversible with use of elevation AND stockings/massage, and fibrotic changes may begin to develop. Further extensive fibrosis and trophic skin changes develop in stage III; these changes are consistent with elephantiasis.

Which approach uses synergistic patterns in training in an attempt to improve motor control through central facilitation in post stroke therapy?

Brunnstrom

Which nerve root is most commonly affected post-operatively in cervical decompression?

C 5 The C5 nerve root is the most frequently involved root after cervical decompression. Surgery. The involvement of other cervical nerves (C6, C7, or C8) has also been reported. Among the proposed mechanisms for the development of C5 palsy, the most acceptable etiologies have been reperfusion injury of the spinal cord after decompression of a chronic compressive lesion of the cervical cord or the tethering effect of the nerve root secondary to posterior drift of the spinal cord after cervical decompression

A 44 year-old female presents with neck pain radiating down the right arm. MRI reveals an acute C6-C7 disc herniation. Which nerve root is most at risk for compression?

C 7

What prescription is best for lateral column overload?

Reverse Mortons

what cervical vertebral level exhibits pseudosubluxation in half of almost all pediatric patients

C2 on C3

The most common level of cervical facet joint mediated pain is?

C2-C3

you suspect someone has fibromyalgia, what labs should you get to rule out other diagnosis

CBC, C reactive protein, and ESR

A 71-year-old man with metastatic prostate cancer to bone on androgen deprivation therapy presents to your office. He has clinically stable disease confirmed by recent computed tomography of the chest, abdomen, and pelvis. Magnetic resonance imaging of the spine demonstrates mild, diffuse degenerative disease. He has developed bilateral upper and lower extremity paresthesias and progressive gait dysfunction over the past several months. Which of the following is the most likely cause of his symptoms?

CIDP

2/3 of infections that occur 1- 6 months post heart transplant are caused by?

CMV

36 year old man with a history of substance use disorder an dunexplained weight loss presents with symmetric numbness and burning pain in his feet bilaterally, which has been gfrradually worsening over teh past 2 years. HIV positive what infectious agent is the most likely cause of his neuropathy?

CMV? in immunocompromised patients, CMV directly infects peripheral nerves causing inflamation and necrosis. CMV neuropathy may be evident i npateitns with CMV and avanced AIDS for example. THe classic manifestation of CMV neuropathy is a mononeuropathy multiplex characterized by painful, stepwise, multifocal sensorimotor deficits. It may be rapidly progressive with nerve injury due to primarily axonal degenration, although segmental demyelination may also present. CMV may also result in a causa equina syndrome, wiht lower extremity weakness and numbness and sphincter dysfunction due to axonal injury

WHat are you screening for when you ask an older patient to draw a clock?

COgnitive problems

what electrodiagnsotic technique can be used to study proximal segments of teh median nerve specifically without artifact from co activation of teh ulnar enrve?

COllision technique

modalities such as whirlpool baths and fluidotherapy work to alleviate pain through? conduction conversion evaporation convection

COnvection

Formula for CPP (cerebral perfusion pressure)

CPP = Mean arterial pressure minus intracranial pressure

compare mixed across wrist ( palm diff) D 4 comparison and ( ring diff) D 1 comparison ( thumb diff) add these differences together and what difference will be positive for CTS??

CSI ( combined sensory index) greater than 0.9 is spostive for CTS

what terms describes when a person has a complex metabolic syndrome associated with underlying illness which is characterized by underlying illness which is characterized by loss of muscle with or withoutloss of fat mass?

Cachexia

The patient is interested in trialing a medicine his friend recommended, topical capsaicin, but wants to know more about its mechanism of action; your attending asks you to explain the mechanism of action to the patient. You say?

Capsaicin depletes substance P, thereby decreasing pain transmission It is important to not only understand the indications for certain medications, but also how they work. Capsaicin acts by depleting substance P from nerve endings; substance P is a neuropeptide that has many roles, one of which is functioning as a neurotransmitter and modulator of pain. Carbamazepine's mechanism of action is inhibition of sodium channels on neurons. L type calcium channel blockade in the central nervous system is the mechanism of gabapentin (and pregabalin, which is structurally very similar). Inhibition of PLA2 enzyme is accomplished by corticosteroids.

Which anticonvulsant also acts as a mood stabilizer

Carbamazapine

Which anticonvulsant medication is most likely to cause bone marrow suppression?

Carbamazapine

which anticonvulsant is likely to cause bone marrow suppression?

Carbamazapine

The equation for cardiac output is which of the following? HR: heart rate. SV: stroke volume.

Cardiac output = HR x SV. This is the amount of blood that can pump through the heart in a given cardiac cycle.

Three weeks after a severe traumatic brain injury (TBI) with basilar skull fracture, a 23-yearold patient in a rehabilitation unit develops new onset ocular pain, unilateral proptosis, and unilateral orbital vascular congestion. What is the most likely diagnosis?

Carotid cavernous fistula

Patient with SCI injury near L1, significant pain, motor weakness of the lower extremities, loss of bulbocavernosus, ankle , plantar and anal reflexes. Where is the lesion?

Cauda Equina -pain

How does choroid plexus papilloma affect CSF?

Causes hydrocephalus through increased secretion of CSF

Which of the following NSAIDS does not inhibit platelet aggregation? Celecoxib? keotrolac? Diclofenac? Aspirin?

Celecoxib

The most common location for metastatic brain tumors is

Cerebral hemisphere

On trauma imaging, he is found to suffer from a T12 spinous process fracture that extends all the way through the spinal column and into the vertebral body anteriorly. This is known as what kind of fracture, and patients often demonstrate what neurologic findings?

Chance fracture, normal neurologic exam

If patient has anterior dislocation of sternoclavicualr joint but has a normal x ray what test should you do next?

Chest CT

A child has been diagnosed with SMA. He is able to sit independently but has never been able to stand or walk. This level of function is most consistent with SMA type:

Classically, children with SMA type 1 never achieve independent sitting. Those with SMA type 2 are able to sit but not stand or walk. Standing and walking are achieved by those with SMA type 3 and 4, with later age of symptom onset in type 4.

Clavicle fractures most commonly occur at which portion of the clavicle?

Clavicle fractures most commonly occur within the middle ⅓ of the clavicle.

Characteristic neuromuscular manifestations of uncontrolled hypothyroidism include

Clinical manifestations of uncontrolled hypothyroidism include muscle weakness and pain. Polyneuropathy attributed to hypothyroidism is typically a demyelinating neuropathy that favors motor neurons. Upper motor neuron signs are not observed.

A 5 year-old boy with spina bifida has full hip flexion against gravity and his knee extension strength is at least 4/5. The child has ankle dorsiflexion but not plantarflexion. His feet are in neutral position. Hip examination is symmetric. What will most likely be this child's primary means of mobility as he grows older?

Community ambulation without assistive device This child has a strong quadriceps muscle and no deformities noted at 6 months of age. He is reported to be healthy. Sitting balance and neurologic level were good predictors of ambulation potential. The best early predictor of ambulation in children with spina bifida is a strong quadriceps muscle. Negative predictors are spine and lower extremity deformities and obesity. Children do not typically learn to use crutches until 3 to 5 years of age or older.

During gait observation, you notice that during stance phase of the left leg, the patient leans their torso laterally over their left leg. Which of the following muscles should be strengthened in order to treat this problem?

Compensated trendelenburg gait involves the patient essentially throwing their torso laterally over their weak leg (the left leg in this case) to compensate for the hip abductor weakness. Thus, to treat this abnormality, the gluteus medius of the leg being stood on during the gait abnormality should be strengthened. In practice, it is wise to strengthen the bilateral hip girdle musculature for muscular symmetry.

Which of the following is associated with clinically identifiable peripheral nerve damage?

Complex regional pain syndrome (CRPS) type I (also known as Reflex Sympathetic Dystrophy) generally develops after minor trauma or surgery and does not involve a clinically identifiable peripheral nerve injury. CRPS type II, previously known as Causalgia, is associated with damage to a major nerve. The sympathetic nervous system is also thought to play a role in the pathophysiology of CRPS, however it is not clearly understood. Sympathetic and catecholaminergic dysfunction may be responsible for autonomic manifestations in CRPS. The exact cause of phantom pain is unclear, but it appears to originate in the spinal cord and brain. In myofascial pain syndrome, sensitive areas in tight muscles, or trigger points, can cause strain and pain throughout the muscle and referred area.

The decrease in cardiac mortality from participation in a program of cardiac rehabilitation is:

Comprehensive cardiac rehabilitation programs that address reducing risk factors and lifestyle changes such as nutrition, weight loss and smoking cessation have an impact on cardiac mortality by reducing risk by 25%.

nerve injury at Arcade of Frohse results in?

Compression by the Arcade of Frohse can cause posterior interosseous neuropathy (PIN-opathy) - a pure motor neuropathy that would spare brachioradialis (not a PIN-innervated muscle) and show normal radial SNAPs. "Normal SNAPs to digits 1,2,5, snuffbox. Normal EMG to BR. Abnormal EMG to EIP"

16 year old male baseball player presents with a one day od right arm pain and swelling. He denies trauma and reports that tha pain developed gradually over thecourse of the practice to the point where he stopped playing. He has no significant past medicl history or family history. physical examination is remarkable for visibly dilated veins and cyanosis of the right arm. What is the most appropriate diagnostic test to obtain in this patient.

Compression ultrasound fo the right arm patient has an upper extremity DVT known as Paget Schroetter syndrome. This is an axillary subclavian vein thrombosis associated with the strenuous and repetative activity of the upper extremities. the most appropriate diagnostic test is a compression ultrasound odf the affected extremity, but the gold standard test contrasts venography

A patient with aphasia demonstrates intact fluency and comprehension, but cannot repeat phrases. What type of aphasia does this patient have?

Conduction aphasia involves a lesion to the arcuate fasciculus of the left cerebral hemisphere, and produces impaired repetition with intact fluency and comprehension. Transcortical motor aphasia involves impaired fluency, but intact comprehension and repetition. Transcortical sensory aphasia involves impaired comprehension, but intact fluency and repetition. Anomic aphasia involves an essentially normal language output and comprehension, except the patient has difficulty naming objects and other nouns.

Which is associated with multidirectional instability of the shoulder? Congenital hyperlaxity of the glenohumeral joint or acute dislocation of the shoulder

Congenital hyperlaxity of the glenohumeral joint

Juvenile idiopathic arthritis (JIA) can be differentiated radiographically from ankylosing spondylitis (AS) by the:

Constellation of cervical facet joint ankylosis and vertebral body hypoplasia in patients with JIA By definition, JIA affects the immature, growing skeleton. Diffuse ankylosis of the cervical facet joints in the setting of vertebral body and disc hypoplasia is virtually diagnostic. In contrast, AS affects the mature skeleton after the intervertebral discs have formed. The cervical spine is more commonly involved in patients with JIA than other spinal levels with AA instability being the most common radiographic finding. Enthesopathy and tarsal disease in the first year of presentation is diagnostic for JIA even in the absence of axial disease. Klippel-Feil syndrome may be difficult to distinguish radiographically from JIA unless posterior element fusion is a dominant feature in the cervical spine.

What is the initial deformity created in the cervical spine in a whiplash injury?

S-shaped in the sagittal plane

Which of the following muscles is not an elbow flexor? Pronator teres Coracobrachialis Brachialis

Coracobrachialis

In which plane are the thoracic zygapophyseal joints aligned?

Coronal The thoracic zygapophyseal - or facet - joints are aligned in the coronal (frontal) plane. The cervical facet joints are aligned in the coronal oblique plane. The lumbar facet joints are aligned in the sagittal oblique plane.

Which of the following antineoplastic agents is LEAST neurotoxic? Bortezomib Cyclophosphamide Paclitaxel Lenalidomide

Cyclophosphamide is not generally neurotoxic. Bortezomib, paclitaxel, and lenolidomide are all neurotoxic.

In Clay shoveler's fracture, what anatomic region of lower cervical and upper thoracic spine is affected?

SPINOUS PROCESS

Which of the following factors increases the likelihood of employment after spinal cord injury?

Single pre-injury marital status

which muscle fibers use aerobic metabolism? anerobic metabolism? both? Type1, Type 2 a, Type 2b

type 1 - slow and use aerobic type 2 a fast and use both aerobic and anaerobic type 2 b- fast and use anaerobic

WHAT IS LIKLEY TO NOT HAPPEN AFTER FINSIHING CARDAIC REHAB INCREASE QUALITY OF LIFE DECREASE RISK OF RECURRENT MI INCREASE CHANCE OF GOING BACK TO WORK

DECREASE RISK OF RECURRENT MI

Duchenne muscular dystrophy (DMD) usually begins with weakness in which of the following muscle groups?

DMD's earliest signs of muscle weakness are classically found in the neck flexors.

What are the 11 criteria for systemic lupus erythematous?

DOPAMINE RASH

A resting wrist-hand-orthosis (WHO) has which of the following characteristics:

Distal and proximal interphalangeal joint extension

75 year old woman with osteoporosis fell on her outstretched hand and presents with wrist pain. what bone is fractured?

Distal radius fracture

The most effective nonsurgical treatment for de Quervain's tenosynovitis is:

Corticosteroid injection alone

A 50 year-old male is admitted following a motor vehicle accident (MVA). He is diagnosed with a traumatic brain injury (TBI). On MRI of his brain you note focal changes involving the medulla. What grade of diffuse axonal injury (DAI) would you assign to this patient?

DAI is graded on MRI criteria. A simplified grading method is as follows: Grade 1 = no focal changes on MRI. Grade 2 = focal changes on MRI. Grade 3 = brainstem involvement.

A 50 year-old male is admitted following a motor vehicle accident (MVA). He is diagnosed with a traumatic brain injury (TBI). On MRI of his brain you note focal white matter changes involving the medulla. What grade of diffuse axonal injury (would you assign to this patient.

DAI is graded on MRI criteria. A simplified grading method is as follows: Grade 1 = lobar white matter changes on MRI. Grade 2 = central white matter (corpus callosum) changes on MRI. Grade 3 = brainstem involvement.

While performing a nerve conduction study (NCS) you apply current to the nerve and detect a small amplitude. You then move distally on the limb and apply current again to the nerve. To your surprise, the amplitude is normal. What is the most likely explanation for this finding?

Decreased amplitude throughout an entire nerve's length suggests axonal loss. EMG will show decreased recruitment in these cases. Conduction block is seen with proximally reduced amplitude, but stimulating distally beyond the spot of conduction block can demonstrate a normal "repaired" amplitude.

What is an indication of success in an exercise training in a pulmonary rehabilitation program for patients with COPD includes which of the following?

Decreased dyspnea at rest and exertion

What would be some good prescriptions for medial column overload

Deep heel cup, semi rigid plate, or wide plate

THe --- Reflex may be the first of all reflexes to return , occurs within hours or days following SCI, and shows a high correlation with complete injuries and a poor prognosis for lower extremity motor recovery and function?

Delayed planter The delayed plantar reflex may be the first of all reflexes to return. Occurring within hours or days following SCI. Elicitation of this reflex is similar to that of the babinski sign but for deep pressure rather than a light stimulus. High correlation with complete injuries. If persisting for longer than 2 days, it is associated with a poor prognosis for lower extremity motor recovery and function.

Measurement of F-wave minimal latencies are a useful part of the electrodiagnostic workup for:

Demyelinating neuropathy F-wave minimal latency is notably prolonged (or absent) in demyelinating neuropathies such as acute and chronic inflammatory demyelinating polyradiculoneuropathy (AIDP, CIDP), and is among the electrodiagnostic criteria for demyelination

A 75-year-old patient was treated for local prostate cancer 15 years ago. He has received leuprolide (Lupron) for chemical castration since his diagnosis. He is currently asymptomatic with an undetectable prostate specific antigen and requests guidance in developing a comprehensive exercise program. Which study would be most useful in counseling this patient?

Dexa Chemical castration to reduce circulating testosterone levels places prostate cancer patients at risk for the development of osteoporosis. Given this patient's age and the extended interval of Lupron therapy, he is at risk for osteoporosis and should be screened before beginning exercise. Dual energy x-ray absorptiometry scan is the most sensitive of the studies listed for detecting osteoporosis.

In a 45-year-old man with a spinal cord injury (SCI) and a history of coronary heart disease, which risk factor is an indication for the most stringent control of low-density lipoprotein (LDL)?

Diabetes

The term diaschisis refers to which of the following?

Diaschisis in TBI patients refers to a TBI causing a "split across" type of injury. In this case, the TBI damages one region of the brain, and through neural network connections between this region and a distant brain region, the distant brain region also becomes damaged and dysfunctional. Even though the distant region was not directly involved in the trauma, its strong connections with the primary region of injury make it susceptible to injury if this primary region is traumatically damaged.

Diazepam's mechanism of action is best described as which of the following?

Diazepam is a GABA-A agonist, which increases presynaptic chloride influx into the axon, thus inhibiting action potential transmission; this is useful for reducing spasticity. Baclofen is a GABA-B (1 and 2) agonist, and dantrolene is a ryanodine receptor antagonist. All of these medications serve to decrease spasticity.

The most common site of maximum tenderness with a navicular stress fracture is:

Dorsally between the tibialis anterior and the extensor hallucis longus tendons

In patients with parkinsons planned for awale deep brain stimulation, what drug shoudl be avoided?

Droperidol Droperidol antagonizes D 2 dopamine receptors. This precipitates or exacerbates parkinsons like phenothiazines and metoclopramide.

Which of the following medications is FDA-approved to treat diabetic peripheral neuropathic pain? Duloxetine Amitriptyline Tramadol Gabapentin

Duloxetine and pregabalin are FDA-approved to treat pain due to peripheral neuropathy in diabetes.

What is the strongest predictor of a seizure-related motor vehicle crash?

Duration of seizure free interval Seizures are responsible for 1% of all motor vehicle collisions. Minimum requirements for length of seizure-free time before a person can drive vary from state to state but range from 3-12 months. The strongest literature confirmed predictor of a seizure related motor vehicle collision is duration of seizure free interval. A 12-month seizure free interval prevents 80% of crashes. A 3-month interval prevents 50% of crashes. Other risk factors for motor vehicle collisions in individuals with seizures include age, gender, generalized or complex seizures, partial complex seizures, history of multiple seizures, and noncompliance with antiepileptic medications. The probability of a seizure related collision is decreased by a long seizure free period, reliable aura, and the use of the least sedating, but most effective, medication.

What congenital disease is caused by a mutation on chromosome 2, of autosomal recessive inheritance. Although the function of the protein is unknown, its deficiency leads to limb girdle weakness. Onset is typically in the teenager or early adult years.

Dysferlin deficiency

A 35-year-old man who sustained a severe traumatic brain injury (TBI) 48 hours ago has now developed new tachycardia, hypertension, agitation, and hallucinations. What is the most likely diagnosis?

ETIOH withdrawal

The Brunnstrom method of rehabilitating a stroke patient aims to do what?

Encourage primitive reflexes and flexor synergy patterns to "use what the patient has"

Which attribute distinguishes end plate activity from fibrillation potentials? End plate spikes have initial negative phases Fibrillation potentials are less than 1.5 msec in duration End plate spikes fires regularly Fibrillation potentials are usually less than 150 microvolts

End plate spikes have initial negative phases End plate activity is generated when the needle is close to the motor end plate thus producing an initial negative phase since the potential is moving away from the recording electrode. End plate activity fires irregularly whereas positive sharp waves and fibrillations discharge regularly. The end plate spikes are fast and occur in a series. Fibrillations are slow at less than 5 Hz and 1-5 msec in duration and or 20- 200 µV in amplitude.

A 15 year-old obese male develops sudden-onset left hip groin pain. He walks with a trendelenburg gait. X-rays reveal 36% slippage of the femoral metaphysis under the epiphysis. Which of the following is important to screen for in this patient?

Endocrine Adolescent obese males are at greatest risk for slipped capital femoral epiphysis (SCFE), in which the metaphysis of the femur slips under the epiphysis (like ice cream falling off a cone). These patients require orthopedic surgery (ORIF) and screening for endocrine abnormalities such as growth hormone deficiency or thyroid disease. SCFE is graded on the % slippage that occurs. Grade 1: <33% slippage. Grade 2: 33-50% slippage. Grade 3: >50% slippage. There is no Grade 4.

Objectives of the Joint Commission include:

Eval of safety standards Joint Commission Disease-Specific Care Certification was established in 2002, and today has been achieved by more than 2,000 clinical care programs nationwide. Certified programs must demonstrate a systematic approach to care delivery and a commitment to performance improvement through ongoing data collection and analysis. The discipline and self-assessment inherent in meeting these requirements creates a road map to improving patient care quality. Licensure falls under the jurisdiction of state and federal governments

Endoskeletons weight less than exoskeletons

Exoskeletal prostheses are more rugged, require less maintenance, cannot be adjusted for alignment after fabrication, and can accommodate only a restricted number of foot and knee units. Furthermore, these prostheses tend to weigh more than the equivalent endoskeletal prostheses. For these reasons, exoskeletal prostheses are prescribed less often than endoskeletal prostheses. Endoskeletal prostheses are modular in design, allowing relative ease of adjustment of alignment and replacement of parts. They are also easier to suspend by virtue of their relatively lighter weight.

What is a pure motor late response which occurs after teh compound motor action potential?

F wave

Normal forced expiratory volume in 1 second (FEV1) decreases by approximately how many cc/year?

FEV1 normally decreases by 30 cc/year. In smokers it can be over double this rate.

Which of the following types of exercise places the greatest stress upon a tendon, leading to highest risk for tendon rupture? concentric or essentric? Fast or slow?

Fast essentric

Which of the following is the most common lower limb congenital limb deficiency?

Fibular hemimelia in general is the most common lower limb congenital limb deficiency. It involves the fibula bone itself being absent. PFFD involves a short femur. Patients typically also have a fibula deficiency, and a surgical procedure called a Van Ness Rotation is sometimes performed, involving rotating the foot around so that the ankle is effectively the new "knee", and the patient can be fitted for a lower limb prosthesis for ambulation.

In order to differentiate a lesion of the common peroneal (fibular) nerve from the deep peroneal (fibular) nerve at the fibular head, which muscle should be tested with needle electromyography? A. Long head of the biceps femoris B. Short head of the biceps femoris C. Peroneus (fibularis) longus D. Tibialis anterior

Fibularis longus

Which of the following maneuvers describes the Jerk test with respect to the shoulder?

Flexing the shoulder to 90 degrees, internally rotating, and adducting across midline with a posterior axial force describes the Jerk test, which is a test for posterior shoulder instability. The Kim test (flexing shoulder beyond 90 degrees and applying posteroinferior force) is also a shoulder posterior instability test. Abducting, externally rotating, and applying anterior force describes the anterior apprehension test, which unsurprisingly tests for anterior shoulder instability.

You are caring for a patient after traumatic brain injury who presents with hyponatremia. Labs reveal elevated urine osmolality and decreased serum osmolality. What is the most appropriate treatment for this patient?

Fluid replacement with normal saline This patient has cerebral salt wasting (CSW). Fluid restriction is the treatment for SIADH while fluid replacement with normal saline is the treatment for CSW. Desmopressin (DDAVP) is used in Diabetes insipidus. Mannitol is used for elevated intracranial pressure after TBI.

Treadmill training in Parkinson's Disease patients improves:

Gait speed

Which organ system is responsible for the leading cause for rehospitalization for medical complications after a traumatic spinal cord injury?

Genitourinary Overall, diseases of the genitourinary system such as urinary tract infection are the most common reason for rehospitalization, followed by diseases of the skin (pressure ulcers), then respiratory and musculoskeletal systems. Respiratory diseases were more likely associated with hospital admission in persons with tetraplegia, while diseases of the skin were more likely associated with rehospitalization in those with paraplegia.

Botulinum toxin injections should not be performed when a patient is taking which of the following, due to potentiation of the botulinum toxin?

Gentamycin Aminoglycoside antibiotics can potentiate the effect of botulinum toxins and botulinum toxins should not be utilized for patients taking this class of antibiotic. Other medications that can potentiate the effect of botulinum toxins include quinidine, anticholinergics, and curare-like compounds.

54 year old male presents with pain in the toes. X ray shows juxta articular erosion with an overhanging edge. There is soft tissue tophus with calcifications. what is this image consistant with?

Gout

Slipped capital femoral epiphysis (SCFE) involves sudden-onset hip/groin pain due to the femoral head sliding off the growth plate, typically in obese adolescent males. X-ray grading is defined by the percentage of slippage of the femoral head off the growth plate. what is the grading for this?

Grade 1: 0-33% sliding. Grade 2: 34-50% sliding. Grade 3: >50% sliding. There is no grade 4. Treatment is orthopedic surgery.

what anti nausea med should be used in a patient with parkinsons to avoid tardive dyskinesia

Granisetron

X ray of a pediatric patient is obtained after an injury in which a bending force was applied to the diaphysis of a long bone. The convex side has a fracture, however there is no fracture on the concave side of the bone. This is what kind of fracture?

Greenstick fracture When a bending force is applied to a bone. THe concave side of the bone undergoes compression of teh cortex, but it remains intact. The convex side undergoes a fracture due to the deforming force.

38 year old mAN WITH hiv PRESENTS WITH PROXIMAL MUSCLE WEAKNESS MYALGIA AND WEIGHT LOSS. cpk IS ELEVATED. What is the most likely cause?

HIV myopathy

What is the formula for VO2 Max?

HR x SV x AVO2 difference

Under the American with Disabilities Act, individuals are considered disabled if they:

Have a record of prior impairment but are not currently impaired

The most common presenting symptom of metastatic brain cancer is:

Headache Headache is the most common symptom (49%), followed by mental disturbance (32%), focal weakness (30%), gait ataxia (21%), seizures (18%), and speech difficulty (12%).

Which patient population meets the criteria for approved payment for cardiac rehabilitation by the Centers for Medicare and Medicaid Services?

Heart failure with ejection fraction of 30%, 7 weeks after discharge from the hospital The Centers for Medicare and Medicaid Services (CMS) made the decision to approve payment for cardiac rehabilitation based on the inclusion criteria of HF-ACTION trial, the largest randomized clinical trial of cardiac rehabilitation (CR) for patients with heart failure (HF) sponsored by the National Institutes of Health. This specifically includes those with symptomatic heart failure with reduced ejection fraction (HFrEF≤35%) who have not been hospitalized for at least 6 weeks and who did not have a planned major cardiovascular hospitalization or procedure in the past 6 months. Though heart failure with preserved ejection fraction (HFpEF ≥45%) is the most common type of HF among older adults, such patients are usually not covered for CR by CMS due to lack of evidence.

A 34 year-old female complains of pain across her upper back. You diagnose her pain as myofascial in etiology. You recommend trying conversion thermotherapy in the form of which of the following? Ultrasound Hot water whirlpool Cold pack Heat pack

Heat packs and cold packs are a form of conduction thermotherapy. A whirlpool is a form of convection thermotherapy which utilizes gross molecule movement to transfer heat. Conversion thermotherapy involves using a source of one type of energy which is converted to heat, such as using ultrasound (sound waves) to heat deep tissues.

ITBS has been associated with what biomechanics of the hip and knee?

ITBS has been associated with greater peak hip adduction, and with greater peak knee internal rotation angle. Foot and ankle mechanics have not yet been shown to contribute to ITBS.

treatment of choice in AIDP?

IV immunoglobulin or plasma exchange

Convert IV morphine to oral

IV is 3 times stronger than oral morphine

What is the leading cause of cardiac-related death in younger athletes?

Idiopathic hypertrophic subaortic stenosis. AKA hypertrophic cardiomyopathy Dysrhythmia, coronary artery disease, idiopathic hypertrophic subaortic stenosis and valvular disease all can lead to cardiac-related collapse. Coronary artery disease is the most common cause in the older athlete, and idiopathic hypertrophic subaortic stenosis is the most common cause in younger athletes.

pregnant female with SCI, found to have asymptomatic bacteriuria. how do you advise?

If patient becomes symptomatic then longer antibiotic course

The Fee for Service Model of payment makes separate payments to providers for the services that providers give to beneficiaries for a single illness or course of treatment (episode of care). This model of payment promotes:

Increased service utilization The Fee for Service Model results in increased service utilization since payment is based on how much a provider does, not how well the provider does in treating the patient The Bundled Payments Initiative aims to improve care through a patient-centered approach, emphasizing care coordination and quality. Though limited evidence-based literature regarding bundled payments currently exists, recent data indicates: (1) bundled payments may reduce spending for an episode of care; (2) bundled payments can promote quality improvements in care.

A 31 year-old male is playing tennis. In an attempt to impress his girlfriend, he tries to catch a 130 mph serve with his bare hand. Shockingly, this does not occur as intended, and he develops sudden-onset finger pain. After delivering this blistering serve and declaring that you swung with minimal effort, you rush over to examine his finger. On exam, you notice swelling of the distal interphalangeal joint (DIP) of his index finger. There is pain with varus stress testing of the DIP. X-rays are negative for acute fracture. What is the most appropriate initial step in management? Dynamic varus stress testing under ultrasound Orthopedic surgery referral Immobilization splinting for the DIP

Immobilization splinting for the DIP This patient has sustained a radial collateral ligament injury/sprain/tear of his index finger DIP joint. X-rays have ruled out fracture of the finger/hand, so the most appropriate step is to allow the ligament to heal by using either buddy tape or an extension splint for the finger the prevents flexion, varus, and valgus forces about the DIP. This is not a surgical issue at this point.

. What is the best orthotic management for an acute knee grade 2 medial collateral ligament sprain?

Immobilizer

What is the primary goal of treatment with orthotic bracing for scoliosis from neuromuscular disease?

Improve sitting posture in wheelchair Scoliosis in neuromuscular disease is very common and can begin early and progress quickly. Neuromuscular curve progression is rarely slowed by bracing. Orthoses may be beneficial because they can stabilize a weak trunk, and improve sitting posture in those using wheelchairs. Bracing may cause instability for ambulatory patients however, increasing the risk for falls. Rigid spinal bracing has a negative effect on vital capacity.

A 77 year-old male undergoes a transfer during physical therapy and complains of sudden-onset severe low back pain. X-rays diagnose a T10 vertebral body compression fracture. Which of the following bracing recommendations is most appropriate?

In cases of vertebral body compression fracture, CASH (cruciform anterior spinal hyperextension) or Jewett braces are recommended in order to maintain a neutral (non-flexed) spine position to prevent worsening of the compression fracture via hyperflexion forces. A soft lumbar corset will not provide sufficient stabilizing force to resist spine flexion. A flexion-biased brace is the opposite of what is indicated in these patients. Bracing is not contraindicated in these cases (quite the opposite, as discussed!).

Isometric vs isotonic vs isokinetic exercise

In isometric exercise, no movement occurs throughout the entire exercise. In isotonic exercise, the weight (tone) being applied to a muscle or joint is the same throughout the entire exercise (e.g. biceps curls - the dumbbell weighs the same, and, thus, imparts the same forces onto the arm for the entire exercise). In isokinetic exercise, the movement and rhythm of the exercise is the same throughout the entire exercise (e.g. cardiovascular machines such as the elliptical, cycling, rowing).

In a patient with bilateral knee and finger pain, symmetric joint space narrowing, and positive anti-CCP antibodies, which of the following synovial fluid study results is the most likely?

In rheumatoid arthritis (i.e. this patient) synovial fluid analysis shows increased neutrophils (approximately 75% predominance) with elevated, but under 100k WBC. In septic arthritis the joint fluid may show >75% neutrophils with > 100k WBCs.

A patient requires assistance for transfers. This is defined as:

In simpler terms, impairments, disabilities, and handicaps can be defined as follows. An impairment is essentially a loss of structure or function (e.g. wrist drop). A disability is inability to perform ADLs (e.g. inability to ambulate or perform self care). A handicap reflects the patient's ability to navigate societal constructs (e.g. cannot climb stairs to reach office at work; paraplegia resulting in loss of work as a professional cyclist).

An 82 year-old male sustains a fall, resulting in spinal cord injury. Neurosurgery performs a spinal decompression and fusion. You are consulted on this patient 2 weeks following his injury. On examination you note impaired sensation to light touch and 0/5 strength in bilateral lower extremities. You also note absent Babinski, anal wink, and bulbocavernosus reflexes. Which test may ultimately offer prognostic information in this patient?

In spinal cord injury patients, typically the Babinski reflex returns after 24 hours of initial spinal shock (occurring immediately after injury). Following that, the anal wink and bulbocavernosus reflexes begin to return. In this patient 2 weeks following his SCI, if still none of these reflexes has returned, it may indicate lower motor neuron injury, e.g. cauda equina syndrome. In these cases, EMG of the lower extremities and pudendal nerves is most useful to offer prognostic information on the health of these nerves. In general the optimal time to wait before performing EMG is 4-6 weeks following peripheral nerve injury. Remember that cauda equina syndrome is really a peripheral nerve root injury, and thus, amenable to EMG.

The most common category of spinal cord injury is:

Incomplete tetraplegia According to the National SCI Database, from SCI Model Systems data collected since 1973, incomplete tetraplegia makes up approximately 45% of spinal cord injuries followed by incomplete paraplegia, complete paraplegia, and then complete tetraplegia.

Clinical symptoms and signs of tethered cord syndrome include

Increase in scoliosis The most common clinical signs or symptoms of a tethered cord include spasticity in the lower extremities, decline in lower extremity strength, and worsening scoliosis. Other signs and symptoms that strongly suggest tethering of the spinal cord include back pain, changes in urologic function, changes in gait, changes in sensation, and development of lower extremity contractures.

Leptomeningeal disease is located:

Intradural and extramedullary

Patients with which type of brain injury are most likely to have deep vein thromboses?

Intraparenchymal hemorrhage In a large study, patients with both penetrating (10.5%) and nonpenetrating (6.4%) traumatic brain injury had lower rates of deep vein thrombosis than did patients with nontraumatic brain injuries (16.3%). Among the latter group, patients with brain tumors had the highest incidence (21.2%), whereas those with hypoxic brain injury had 9.6% incidence and those with intraparenchymal hemorrhage had 14.6%.

14 year old girl presents with back pain, worse with sitting, she has left sided scoliotic curve. What conidtion should be ruled out in MRI of the spine?

Intraspinal Syrinx A left sided pattern is associated with a risk for intraspinal syrinx or tumor

How does static streching before running help?

It has NO benefit it is lacking in terms of benefit and may slow sprint sup to 100 meters apreparatory aerobic warm up combined with dynamic range of motion exercvises may be of some benefit for runners ( jumpiong jack, lunges, squats)

In the NOrthern part of the United States, traumatic spinal cord injury occurs with greatest frequency in the month of?

July

Which radiographic findings can be seen in Complex Regional Pain Syndrome?

Juxta-articular osteoporotic changes

6 month old child presents for a well visit. He has a history of scoliosis and a ventricular septal defect. Physical examination is remarkable for a short, broad neck with webbing. He also has a low hairline, the restricted motion of his neck, and a Sprengel deformity. An x ray reveals fusion of C 3 - C7 with hypoplastic vertebrae. What is the most likley cause of his patients symptoms?

Klippel Feil syndrome heterogenous condition associated with congenital failure of segmentation in the developing cervical spine, resulting in fused cervical vertebrae and often hemiverebrae

What is the most common type and cause of TOS?

Neurogenic by compression of components of brachial plexus others: arterial TOS due to ocmpression of teh subclavian artery, venous TOS due to compression of teh subclavian vein, Neurogenic TOS produced by compression of teh brachial plexus

Phantom pain is awhat type of pain? Neuropathic Nociceptive Psychogenic Somatoform

Neuropathic

What lower limb deformity are children with L4 spina bifida most likely to have?

Knee extension contractures The knee extensors (quadriceps) are innervated at the L3-4 level, while the knee flexors (hamstrings) are innervated at the L5-S1 level. A child with L4 preserved level would have quadriceps muscles that work, while hamstrings will either be weak or absent. Foot muscles are innervated at the L5-S2 levels. Equinus and cavus feet result from asymmetric pull of foot muscles, which would be seen in levels of spina bifida L5 and distally.

root level innervation for TFL?

L 4 L5 S 1 superior gluteal nerve

Which of the following provide sensory innervation to the posterior sacroiliac joint?

L 5 Dorsal Ramus The majority of the posterior sacroiliac joint is believed to be innervated by the L5 dorsal ramus and S1, S2, S3 lateral branches. No medial branches are involved in the innervation of the sacroiliac joint.

root level innervation for semimembranosus?

L 5 S 1 S 2 tibial portion of sciatic nerve

What is the most commonly affected level in a degenerative spondylolisthesis?

L4- L5

Lambert Eaton has autoantibodies that target what?

Lambert-Eaton myasthenic syndrome (LEMS) is an acquired disorder resulting from autoantibodies targeting the presynaptic P/Q voltage-gated calcium channels and possibly another presynaptic component (synaptotagmin), leading to a reduction in acetylcholine release.

Deficiency in what results from a mutation of chromosome 6, of autosomal recessive inheritance. Absence of this protein results in hypotonia at birth, severe weakness of the trunck and limbs, and contractures of the feet and hips. Seizures occur in up to 20% of patients. White matter abnormalities may be evident on imaging. A partial deficiency causes a milder phenotype.

Laminin alpha 2

Which type of nerve fibers is responsible for vibration perception?

Large, myelinated sensory fibers Vibration, proprioception, light touch, and deep tendon reflexes are all dependent on large, mye-linated sensory fibers. Small, myelinated and unmyelinated sensory fibers are responsible for pin prick and pain sensation, cold perception, and light touch.

A patient develops lateral knee pain, worse with skiing downhill. He notes that running downhill also causes this pain. On exam, varus and valgus stress testing is negative, as is McMurray, Thessaly, Apley grind, Lachman, posterior drawer, patellar grind, and patellar excursion. There is no tenderness to palpation throughout the knee. He denies stiffness, swelling, numbness, tingling, or pain when going up or down stairs. What is the most likely diagnosis?

Lateral knee pain worse with running downhill or skiing downhill is a classic presentation of popliteus tendonitis. The popliteus muscle unlocks the knee and is located laterally within the knee joint. Treatment involves rest, ice, NSAIDs, and physical therapy.

9 year old boy with limop fo r8-9 months, right thigh pain, right hip stiffness, flexion contracture of right hip, right leg is slightly externally rotated. pain with internal rotation and abduction of right leg. X ray shows a flattened right femoral epiphyses and inferior and lateral displacement of the proximal femur. What is diagnosis?

Leg calve perthes child 4-11 radiographs may be normal

Which of the following is an example of an isokinetic exercise?

Leg press Planks Elliptical machine Biceps curls In isometric exercise, no movement occurs throughout the entire exercise. In isotonic exercise, the weight (tone) being applied to a muscle or joint is the same throughout the entire exercise (e.g. biceps curls - the dumbbell weighs the same, and, thus, imparts the same forces onto the arm for the entire exercise). In isokinetic exercise, the movement and rhythm of the exercise is the same throughout the entire exercise (e.g. cardiovascular machines such as the elliptical, cycling, rowing).

5 year old boy has pain in his right hip, limping, no recent injury or illness, pain with extremes of ROM, no TTP, CBC normal, x ray with small joint effusion with mixed lucency and sclerosis at the femoral head. What is the most likely diagnosis and what is your differential?

Legg Calve Perthes disease Septic arthritis sickle cell anemia slipped capital femoral epiphysis transient synovitis

An 8 year-old male presents with gradual onset right hip pain for the past 1 month. He denies trauma, numbness, tingling, or weakness. He has no past medical history, and has maintained a normal body mass index (BMI). He suffered from an upper respiratory infection (URI) 2 weeks ago, but otherwise has been well. Hip x-rays demonstrate sclerosis of the femoral head. Which of the following is the most likely etiology of this condition?

Legg-Calve-Perthes disease involves idiopathic loss of blood supply to the femoral head, leading to femoral head avascular necrosis (AVN), which can be seen on x-rays as femoral head sclerosis and ultimately femoral head collapse. Treatment may involve observation, rest, bracing, physical therapy, pain control, and/or orthopedic surgery. Slipped capital femoral epiphysis and transient synovitis would demonstrate sudden-onset hip pain, not gradual. This patient has not had trauma, per the question stem, and trauma would more likely cause sudden-onset pain regardless.

Which of the following is the most common pediatric cancer?

Leukemia is the most common pediatric cancer. Astrocytoma and medulloblastoma are the most common solid pediatric cancers (brain tumors). Osteosarcoma is the most common primary bone cancer overall.

Dop responsive dystonia is characterized by a diurnal variation of symptoms. Age of onset is 4-8 years. tone increases throughout the day. often misdiagnosed as cerebral palsy. How do you treat?

Levodopa

Which blood pressure medication classes can be associated with precipitation of gout flare?

Loop diuretics and thiazide diuretics can lead to hyperuricemia and precipitate a gout flare.

What percentage of low back pain has a specific identifiable cause?

Low back pain arises from a number of anatomical structures. In the majority of cases, low back pain cannot be attributed to a specific cause. Approximately 5-15% of cases can be specifically attributed to a case such as an osteoporotic fracture, neoplasm or infection.

In an L 5 radiculopathy which muscle will first have denervation potentials on EMG?

Lumbar paraspinals

diabetes mellitus can cause what kind of plexopathy?

Lumbosacral plexopathy

in CARDIAC REHAB AVOID

MOVEMENTS THAT WOULD CAUSE VALSAVE

A patient presents with progressive, unrelenting low back pain. Two months ago, an uncomplicated diskectomy successfully relieved the patient's radicular symptoms. Which imaging study would most likely confirm the diagnosis at this time? A. X-ray of lumbosacral spine B. Magnetic resonance imaging with contrast C. Ultrasound D. Three-phase bone scan

MRI

A 45 year old women is referred to your clinic as a result of lower back pain and abnormal gait. the patients gait is found to be stiff legged and further observation reveals some spasticity. there is mild to moderate lower extremity weakness with clonus achilles reflex noted on both sides. in addition, there is detrusor hyperreflexia. Which of teh following is th emost liekly diagnosis. MS Herniated lumbar disk infection muscular dystrophy

MS

MVO2 aka cardiac consumption is what formula

MVO2 = HR x SBP HR xSV = cardiac output VO2 Max = CO xAVO2

Which of the following aids in prognosticating outcomes of TBI at 6 months post injury?

Magnetic resonance spectroscopy Magnetic resonance spectroscopy (MRS) provides information about the neurochemical status of the brain and may indicate the extent of brain damage; therefore, it provides important data for evaluating expected outcome. Studies found that MRS studies correlated with outcomes 6 months later. Furthermore, several studies found threshold levels for the metabolites that also correlate with prognosis. Positron emission tomography (PET) uses radioactive tracers that localize to metabolically active regions. PET scanning is more sensitive in detecting abnormalities after traumatic brain injury than is standard neuroimaging, although PET has poor specificity and has not been studied in regard to prognostic value. Earlier studies suggest that the presence of apolipoprotein-E4 allele (APOE-e4) would indicate a worse functional outcome, but that finding has not been replicated. There are conflicting reports on APOE-e4. It is unlikely that a patient's APOE-e4 status will play a role in prognostication at the present time. Transcranial magnetic stimulation is useful in examining cortical motor representation after cerebral injury and may be useful in treatment of TBI but has not been studied in regard to prognostic value.

Regarding the cardiovascular status of a patient after acutely sustaining a spinal cord injury, it is important to?

Maintain a mean arterial blood pressure at or above 85 mm Hg as this is associated with enhanced neurologic recovery

Which wheelchair would be most appropriate for a 50-year-old man who had a stroke with right hemiparesis?

Manual wheelchair with lower seat height

lens dislocation, also known as ectopia lentis is associated with what syndrome?

Marfan syndrome

a 26 year old male has been rushed into emergency room after sustaining a number of gunshot wounds to the trunk and abdomen. Which of teh following procedures are critical in the initial management of acute spinal cord injuries? -Measure forced vital capacity and arterial blood gas -standard radiographic trauma series -avoiding insertion of nasogastric tube -provision of texas or condom catheter

Measure forced vital capacity and arterial blood gas a VC less than 1 liter indicates an extreme ventilatory defect with impending need for assisted ventilation.

A patient suffers a stroke and he is transferred to the stroke rehab unit after medical stabilization. He is seen by physical, occupational, and speech therapies. He has significant aphasia during the initial H&P encounter. You contemplate melodic intonation therapy as a means to help this patient; which type of aphasia below could melodic intonation therapy benefit? Broca aphasa Wernicke aphasia Conduction aphasia

Melodic intonation therapy can be useful for patients with nonfluent aphasia, including Broca's aphasia. The other answer choices are fluent aphasias. Melodic intonation therapy recruits the right hemisphere (speech is typically a left hemisphere activity) by incorporating melodies or rhythms into communication.

Melodic intonation therapy is most useful for which type of aphasia?

Melodic intonation therapy is a speech therapy technique most useful in patients with impaired fluency and repetition, but intact comprehension (Broca aphasia). The patient's fluency is impaired, but by drawing from, and utilizing the abilities of the intact right hemisphere in melody/prosody processing, a patient's fluency can be improved.

Which medication alleviates pain by inhibiting prostaglandin synthesis, is somewhat selective for Cox 2 and has analgesic antipyretic and anti-inflammatory actions?

Meloxicam, Mobic

What are the most common types of fractures in children?

Metaphyseal, epiphyseal, apophyseal

What is an oral analogue of lidocaine used in the treatment of neuropathic pain?

Mexiletine

What type of UE fracture is dewfinitevley treated with functional bracing?

Midshaft transverse humeral fracture

Mirabegron promotes urine storage in the bladder by stimulating which of the following receptors?

Mirabegron is a Beta-3 receptor agonist, which is theoretically very selective for the bladder wall, and promotes urine storage.

A patient who enjoys playing racquetball presents to your office with a trigger finger. You proceed with a corticosteroid injection. What would be your immediate next step in the treatment of this patient?

Modify RAQUET HANDLE A trigger finger is a stenosing tenosynovitis. It develops due to repetitive trauma that causes an inflammatory process to the flexor tendon sheath of the digits. This process forms a nodule in the tendon resulting in abnormal gliding through the pulley system. As the digit flexes, the nodule passes under the pulley system and gets caught on the narrow annual sheath resulting in the finger locked in a flexed position. In an athlete, it often occurs in racquet and club sports. A nodule forms from overuse or direct pressure at the flexor tendon sheath which leads to mechanical catching under the A1 pulley at the MCP level. A corticosteroid injection into the nodule or thickened tendon sheath can improve symptoms in up to 90% of cases. Modification of the racquet handle or grip may facilitate recovery, therefore the immediate next step in the treatment of this patient after a corticosteroid injection would be to recommend the patient modify the grip on their racquet handle. Splinting the finger in extension is not beneficial to the patient. There is no need for referral to the hand surgeon as 90% of patients treated with a corticosteroid injection will benefit from the injection. There is also no need to repeat the injection in 2 weeks as the most immediate next step in treatment would be to modify the causative agent of injury.

A 37-year-old male with Becker Muscular Dystrophy presents for a bracing evaluation. He is noted to have full passive range of motion in all lower extremity joints. Manual muscle testing shows 4/5 initial strength in hip flexors/extensors and knee flexors; 3/5 knee extensors, and flaccid dorsiflexors. The patient ambulates without assistive device for 20 feet with a hyperextended trunk, reciprocal gait pattern, and increased step height. Initial contact occurs through bilateral forefeet, with moderately severe bilateral knee hyperextension at midstance. Which bilateral AFO design will minimize weight, maximize compliance and prevent genu recurvatum?

Molded articulated thermoplastic AFO with a plantarflexion stop set in slight dorsiflexion Although a knee-ankle-foot orthotic would be the ideal choice, most patients with moderate to severe lower extremity weakness find them too heavy, and contribute to overall fatigue. Remembering closed chain kinetics, knee extension will be encouraged by plantarflexion of the ankle, and discouraged by dorsiflexion. The flexibility of a posterior Leaf Spring thermoplastic AFO or spiral AFO, though lightweight and assisting with foot drop, will not prevent this patient's knee hyperextension (genu recurvatum). A solid thermoplastic with anterior malleolar trim lines aligned in dorsiflexion will inhibit the knee hyperextension by promoting knee flexion. If too much dorsiflexion is permitted, his knee may buckle due to knee extensor weakness. Additionally, adjustments in ankle alignment will be difficult to achieve given the solid nature of this brace.

45 year old woman presents with pain and numbness in her left leg and arm. She also developed motor and sensory deficits over the past few days. her pain is in the area of the posterior thighthat is innervated by the sciatic nerve as well as her left forearm in the area supplied by the ulnar nerve. The patient denies any history ofd back pain, falls, or accidents. She denies any sphincter dysfunction or incontinence. What is her diagnosis?

Mononeuritis multiplex. This is a diffuse disorder that is similar to periopheral neuropathy but is distinguished by the involvement of multiple individual nerves. MM is usually the result of sichemic insult to nerves and the most common etiology is small or medium vasculitis. MM occurs in most cases of sytemic vasculitis and maybe its presenting symptoms. MM typically presents with the acute onset of severe pain and numbness in the involved limb, motor and sensory deficits develop over days.

patients with bells palsy will have what on EMG?

Myokymic dsicharge rippling appearnce to overall skin. fascial myokymia can also be due to MS, brainstem neoplasm, polyradiculopathy

Myoplasty involves suturing the muscles to each other, and is technically easier. Myodesis involves suturing the muscles into the bone, produces a more stable surgical result, but is contraindicated in patients with severe dysvascular disease, as it will not heal properly due to the poor blood supply. Myodesis Myoplasty Myotomy

Myoplasty involves suturing the muscles to each other, and is technically easier. Myodesis involves suturing the muscles into the bone, produces a more stable surgical result, but is contraindicated in patients with severe dysvascular disease, as it will not heal properly due to the poor blood supply.

NSAIDs (nonsteroidal anti-inflammatory drugs) reduce pain by reducing the production of which of the following compounds?

NSAIDs inhibit the COX enzyme, which reduces COX's production of prostaglandins. Prostaglandins mediate inflammation. Thus, inflammatory pain is reduced by using NSAIDs.

describe the NYHA cardiac functional classification as NYHA Class 3? NYHA class 1 NYHA class 2? NYHA Class 4?

NYHA cardiac functional classification as NYHA Class III. NYHA Class III patients will be comfortable at rest but otherwise have significant limitations; they can typically perform activities between 2-5 METS (metabolic equivalent). NYHA Class I patients have a cardiac disease that does not limit physical activity including activities >7 METs. NYHA Class II patients are comfortable at rest but ordinary activity causes symptoms, and these patients can perform activities between 5-7 METs. NYHA Class IV patients are unable to carry on any physical activity without discomfort and are usually limited to 2 METS or less.

With Polio, what do sensory nerve studies look like on NCS?

Normal

Which electrodiagnostic finding is most compatible with early stage anterior horn cell disease?

Normal motor conduction studies In anterior horn cell diseases such as amyotrophic lateral sclerosis (ALS), the sensory nerves are not usually affected. In the early stages of the condition motor nerve conductions can be normal. Because ALS is a neuropathic condition with ongoing denervation and reinnervation, one would not expect to find small amplitude motor units with early or increased recruitment; that pattern represents a myopathic process.

Which of the following is the most normal/ideal synovial fluid white blood cell count (WBC)?

Normal synovial fluid WBC should be zero (0) or nearly zero.

In patients with degenerative disc disease of the lumbar spine, what percentage of weight do the zygapophyseal joints bear?

Normal zygapophyeal joints weight bear 33% of the total compressive load. Those with just zygapophyseal arthritis bear up to 47% and those with degenerative disc disease bear up to 70%.

When viewing magnetic resonance imaging (MRI) of the cervical spine, which tissue type is dark on a T1-weighted image and bright on a T2-weighted image?

Nucleus pulposis

A patient with mutations in the survival motor neuron 1 gene (SMN1) would most likely benefit from which of the following medications?

Nusinersen is FDA-approved to treat spinal muscular atrophy (SMA). Riluzole and pyridostigmine are not (they treat ALS and myasthenia gravis, respectively).

A 24 year old male has right hip pain. in training to pass his test for the military he has been running everyday. For the past 2 weeks he has noticed some discomfort in his groin area. Usually the pain is better with rest but worsens as he runs. He has a history of adductor muscel strain last year but no other past issues. He has some discomfort with ambulation. on exam there is mild discomfort on deep palpation of his right grion. He has a normal range of motion of his hip with some discomfort at extremes ranges. His exam findings are otherwise normal, including his genito urinary exam. Pelvis films with a lateral right hip study are normal. WHat is the best next step in management?

OBtain MRI, the patient should be non weight bearing on crutches A stress fracture of the femoral neck is a major cause of groin pain, especially in athletes or military recruits with rapid increases in physcial activity. This involves a gradual onset of groin pain, which poorly localized and aggravated by activity.

List risk factors for plantar fasciitis

Obesity is a risk factor for plantar fasciitis. Plantar fasciitis affects both men and women equally, most commonly between the ages of 40 and 70 years. Factors that increase the tension on the plantar fascia, such as decreased subtalar motion, pes cavus, pes planus, and a tight Achilles' tendon, may contribute to plantar fasciitis.

Which of the following is the most common type of amputation?

Of those listed, transmetatarsal amputations are the most common. Consider that in lower limb amputations, dysvascular disease (the most common cause of lower limb amputations) tends to affect the limb distally > proximally.

You are discussing your patient's new knee-ankle-foot orthosis (KAFO) with the patient's orthotist. She mentions that she intentionally offset the KAFO's hinge joint to sit posterior to the patient's knee. Which of the following is the most likely purpose of offsetting the hinge posterior to the patient's knee? A To promote knee flexion To cause the line of gravity to fall anterior to the knee to offset the anteriorly placed AFO components

Offsetting the hinge joint of a KAFO serves the purpose of moving the line of gravity anterior to the patient's knee, which then tends to lock the knee in extension, producing a stable knee during ambulation that locks appropriately and will not buckle. This is useful in patients with weak quadriceps (not spastic quadriceps) who need extra assistance in extending/locking the knee during ambulation. Promoting knee flexion would occur if the hinge were fixed anteriorly to the patient's knee.

Which of the following juvenile idiopathic arthritis (JIA) subtypes requires ophthalmology referral? RF: rheumatoid factor.

Oligoarticular JIA requires ophthalmology referral, as this subtype is associated with iridocyclitis/uveitis.

The most common organism responsible for septic arthritis in a one-year-old is:

Overall, the most common organism responsible for septic arthritis is Staphylococcus aureus. However, in children ages 2 months to 2 years old, the most common organism is Hemophilus influenza. Neisseria gonorrhea is the most common agent for septic arthritis in sexually active adolescents.

Orientation of ground reaction force vector (GRFV) in sagittal plane during mid stance?

Orientation of ground reaction force vector (GRFV) in sagittal plane during mid stance Normal GRF is located anterior to ankle joint anterior to knee joint posterior to hip joint

A 12-year-old boy presents to you for right ankle pain. His parents brought a recent MRI, and a grade 3 osteochondritis dissecans lesion is noted at the talar dome. You recommend:

Osteochondritis dissecans is a disorder characterized by loss of integrity of the subchondral bone in a focal area of the joint. The injury may be staged with MRI, with conservative measures such as splinting, rest, and physical therapy being valid options for grade I (continuous with underlying bone) or II (partially continuous) lesions; however, grade III (discontinuous but not dislocated) and IV (loose body) lesions require referral to an orthopedic surgeon for consideration of surgical fixation, removal, or other procedures.

You are caring for a patient with a severe burn and malnutrition on the acute rehabilitation unit and looking for an additional agent to support nutrition during his recovery process. Which of the following is an anabolic steroid that is FDA-approved to promote weight gain after severe physiologic stress?

Oxandrolone is an anabolic steroid that is FDA-approved to promote weight gain after severe physiologic stress, such as a burn with hypermetabolic state. Vitamin C is an essential nutrient in wound healing because of its role in collagen formation. Zinc is a stimulant for DNA syntheses and may have a beneficial effect on wound healing. Dronabinol is a cannabinoid medication that is used to treat loss of appetite and weight loss.

Which of the following is optimal initial treatment for congenital hip dysplasia in infants less than 6 months?

PAVLIK The Pavlik harness is the first line treatment. It is a brace which fastens around an infant's legs and connects to the shoulders and torso to prevent mal-tracking of the femoral head within the acetabulum. Spica casting requires a series of hip casting to prevent femoral acetabular dislocation. Abduction braces are generally prescribed after completion of a spica cast series. Closed reduction is not the best initial treatment.

PM&R Recap Home Climate About/FAQ Features & Plans Videos Qbank Oral Boards Cases Mobile App Ambassador Program Contact Us Login | Signup Exam - 5 Questions Pause Exam 1 of 5 Questions: 20% A 23 year-old male sustained an injury while playing soccer 2 weeks ago. He presents for an EMG evaluation due to foot drop. X-rays of the right lower limb are negative. On exam, he demonstrates intact lower limb strength except for 3+/5 right ankle dorsiflexion. Tinel at the fibular head is positive for reproduction of numbness and tingling he is experiencing. Nerve conduction studies demonstrate normal sural sensory nerve action potential (SNAP), prolonged latency of the superficial fibular SNAP, and normal compound muscle action potential (CMAP) of the tibialis anterior (TA) when stimulating below the fibular head. However, when stimulating within the popliteal fossa, the TA CMAP amplitude drops 50% compared to the below fibular head stimulation site. Needle EMG of the TA and extensor digitorum brevis (EDB) reveals decreased recruitment. Needle EMG of remaining muscles is normal. Which of the following is the most appropriate response to these findings?

PM&R Recap Home Climate About/FAQ Features & Plans Videos Qbank Oral Boards Cases Mobile App Ambassador Program Contact Us Login | Signup Exam - 5 Questions Pause Exam 1 of 5 Questions: 20% A 23 year-old male sustained an injury while playing soccer 2 weeks ago. He presents for an EMG evaluation due to foot drop. X-rays of the right lower limb are negative. On exam, he demonstrates intact lower limb strength except for 3+/5 right ankle dorsiflexion. Tinel at the fibular head is positive for reproduction of numbness and tingling he is experiencing. Nerve conduction studies demonstrate normal sural sensory nerve action potential (SNAP), prolonged latency of the superficial fibular SNAP, and normal compound muscle action potential (CMAP) of the tibialis anterior (TA) when stimulating below the fibular head. However, when stimulating within the popliteal fossa, the TA CMAP amplitude drops 50% compared to the below fibular head stimulation site. Needle EMG of the TA and extensor digitorum brevis (EDB) reveals decreased recruitment. Needle EMG of remaining muscles is normal. Which of the following is the most appropriate response to these findings? A MRI is recommended B Ultrasound-guided intervention is recommended C Surgical opinion should be sought D Prognosis for recovery is good Explanation: This patient's results demonstrate neurapraxia of the common fibular nerve, likely due to some physical injury sustained during the soccer match. Neurapraxia can be differentiated from axonotmesis (axon loss due to crush/stretch injury) and neurotmesis (axon loss due to complete nerve transection) due to the normal CMAPs present below the level of injury at 2 weeks out from injury. There is also the lack of active denervation (fibrillations and positive sharp waves) that suggests neurapraxia rather than axon loss. Neurapraxia, you recall, manifests as a conduction block, as in this case (normal amplitude CMAP distal to the lesion, decreased amplitude CMAP proximal to the lesion). Neurapraxia/conduction block represents a focal demyelination event; thus the axons are intact, and all that is required is remyelination for the CMAP, recruitment, and patient's strength to recover. If this were axon loss, the CMAP ampli

A collegiate runner presents to you with pain overlying the tuberosity of the 5th metatarsal. You are concerned for a possible Jones fracture, but X-ray and MRI are both negative for fracture. Tendonitis of which muscle is the likely culprit for pain overlying the tuberosity of the 5th metatarsal?

Pain along the lateral aspect of the foot, specifically overlying the base or tuberosity of the 5th metatarsal, should prompt consideration of fibularis brevis tendinopathy. This tendon inserts onto the 5th metatarsal tuberosity. This is the same anatomic location that should alarm clinicians to the possibility of a Jones fracture. The fibularis longus tendon inserts onto the medial cuneiform and 1st metatarsal bone. The tibialis anterior tendon inserts onto the medial cuneiform bone and the base of the first metatarsal bone. The tibialis posterior tendon inserts primarily onto the navicular bone.

Which of the following spinal cord injury levels is most likely to necessitate ventilatory support?

Patients with cervical spinal cord injury (SCI) are at especially increased risk of respiratory insufficiency, chiefly due to diaphragm denervation. The phrenic nerve controls the diaphragm (the body's chief muscle of inspiration), and is innervated by C3, C4, C5 nerve roots. Thus, a patient with a C6 level SCI would be expected to still possess complete voluntary diaphragm control. However, patients with C3/C4/C5 injuries may have loss of diaphragm function secondary to phrenic nuclei injury within the spinal cord. The higher the injury in the spine, the more nervous tissue in the inferior (distal) spinal cord will be affected. Thus, the higher the level of injury (C3 in this case), the most loss of diaphragm control will take place.

A 27 year-old male is running a marathon when he trips and lands on his right shoulder. He experiences immediate severe shoulder pain. On exam, there is asymmetry when comparing his right shoulder to his left. You diagnose an anterior shoulder dislocation. This patient's ultimate treatment will most likely be which of the following?

Patients with traumatic unilateral shoulder dislocations typically require surgery.

Which of the following juvenile rheumatoid arthritis (JRA) subtypes requires ophthalmology referral? RF: rheumatoid factor.

Pauciarticular JRA requires ophthalmology referral, as this subtype is associated with iridocyclitis/uveitis.

Which peripheral nerve innervates the bladder to stimulate bladder emptying via detrusor contraction?

Pelvic "Parasympathetics Pee Pelvic": the pelvic nerve carries parasympathetic fibers which innervate the detrusor muscle, which then contracts to cause bladder emptying. Credit to Dr. Amanda Harrington for this great mnemonic!

Which massage technique has as its goal the breakup of tissue and muscle adhesions?

Petrissage

The modality of utilizing sound waves to push medication through the skin and deeper tissues in order to reach a target structure is known as:

Phonophoresis is the correct answer. Iontophoresis is the concept of using electricity to drive ions through the skin towards a target structure. Magnetophoresis and transdermophoresis are not actual modalities.

What is the most common cause of death in patients with ventilator-dependent tetraplegia who have survived the first 24 hours?

Pneumonia

A patient presents to you for evaluation for left leg weakness. History is significant for fever, headache and myalgias a few days prior to the acute onset of nonprogressive flaccid paralysis of left lower leg. Patient denies any difficulty with speech or swallow. Exam is significant for weakness in left lower leg, muscle atrophy, decreased tone and areflexia. CSF shows elevated proteins and pleocytosis. MRI shows increased T2 signal in the region of ventral horn of spinal cord. EMG shows multifocal denervation. What is the likely diagnosis?

Polio

You are performing serial EMGs on a patient. Over time you notice that while he used to demonstrate polyphasic motor unit action potentials (MUAPs) in a particular muscle, on his most recent EMG these polyphasic potentials have been replaced by larger, non-polyphasic potentials. What is the most reasonable explanation for this finding?

Polyphasic potentials represent ongoing reinnervation due to collateral sprouting of existing axons. The polyphasic property arises due to non-uniform myelination of these collateral sprouts during this early reinnervation stage. Once myelination is complete, the motor unit's branches all conduct the action potential at essentially the same speed, producing a nice, uniform "roller coaster" bump (uniform-appearing MUAP) rather than the polyphasic, serrated potential seen in unmyelinated collateral sprouts

What is an advantage of intermittent positive pressure breathing (IPPB) devices for patients with postpolio syndrome?

Portability

Which of the following findings on an electrodiagnostic study would help diagnose Myasthenia Gravis? Increment at baseline Decrement immediately after exercise Post-activation exhaustion 3 minutes after exercise Post-activation potentiation immediately after exercise

Post-activation exhaustion 3 minutes after exercise Post-activation exhaustion refers to the smooth decrement of at least 10% that occurs 2-4 minutes after exercise as a feature of Myasthenia Gravis (MG). At baseline, there is no increment in neuromuscular junction disorders. Post-activation potentiation refers to the increment after exercise or high frequency repetitive stimulation in Lambert-Eaton Myasthenic Syndrome. Immediately after exercise there is a repair of the baseline decrement in MG.

Concentric activation of the iliopsoas muscle is essential to which phase of the gait cycle?

Pre swing

A patient with a known history of chronic lymphocytic anemia and liver cirrhosis secondary to alcohol abuse presents with painful peripheral neuropathy. Which of the following antiepileptic medications is least likely to cause adverse effects in this patient?

Pregabalin Based on practice guidelines from AAPM&R, AAN and AANEM, pregabalin is supported by level A evidence for managing painful peripheral neuropathy. Pregabalin has no organ toxicities and is generally well tolerated. Carbamazepine is associated with aplastic anemia and hepatic toxicity. Valproate is associated with hepatic and hematologic toxicities. Phenytoin is associated with neurotoxicity.

with fibular nerve study, a small CMAP distally could indicate what normal anomaly

Presence of accessory peroneal nerve

Which risk factor is most clearly linked with developing subsequent osteoarthritis of the knee?

Prior injury of the knee is the single most important modifiable risk factor for men and second for women for subsequent development of knee osteoarthritis. even more than age

Which therapy approach uses spiral and diagonal components of movement rather than the traditional movements in cardinal planes of motion with the goal of facilitating movement patterns that will have more functional relevance than the traditional technique of strengthening individual group muscles

Proprioceptive neuromuscular facilitation

Which of the following mediators is released from macrophages and neutrophils that act as vasodilators and increase microvascular permeability after burn injury?

Prostaglandins Prostaglandins are released from macrophages and neutrophils and act vasodilators increase microvascular permeability. There are a few inflammatory mediators released during thermal injury. Histamine causes an increase in arteriolar dilatation and tissue pressure, leading to increased microvascular permeability. Thromboxane is produced by platelets. There is minimal effect on vascular permeability. Catecholamines cause arteriolar vasoconstriction.

A 74 year-old female falls onto her outstretched hand and sustains a proximal humerus fracture. On xrays, you notice that the humerus is intact except for the greater tuberosity which is displaced. What is the next best step?

Proximal humerus fractures are classified using the 4-part classification system. In 1-part fractures, the humerus remains as one whole part (intact), with no elements displaced off the humerus. These are treated with a sling. In 2-part fractures, one of the 4 "parts" (greater and lesser tuberosities, humeral head, humeral shaft) is displaced from the other 3, thus leaving 2 "parts" of bone existing in the proximal arm: the displaced fragment and the still intact other 3 parts. In 3-part fractures, 2 of these parts are broken off, and in 4-part fractures, all 4 of these parts are separated from one another. 2-4 part fractures require surgery, as in this question's case.

A 75-year-old female presents to clinic with a large, cool synovial effusion of her right shoulder. Physical examination is significant for pain-limited range of motion (ROM) and profound rotator cuff muscle weakness. X-rays of the patient's shoulder reveal superior subluxation of the humeral head, extensive bony destruction of the glenohumeral joint, soft-tissue effusion, and calcific deposits. The most likely diagnosis is:

Pseudogout The patient presents with a classic "Milwaukee shoulder" - MSS is a destructive, calcium phosphate crystalline arthropathy; it encompasses an effusion that is noninflammatory with numerous aggregates of calcium hydroxyapatite crystals in the synovial fluid, associated with rotator cuff defects . Calcium hydroxyapatite crystal disease is characterized by recurrent painful periarticular calcific deposits in tendons, soft tissues, or intra-articular surfaces. The pathophysiology is believed to be the intra-articular calcium hydroxyapatite deposition. MSS occurs in elderly patients typically aged 60-90 years. There is a female preponderance in the ratio of 4 : 1. Radiographic changes on plain X-ray show joint space narrowing, subchondral sclerosis with cyst formation, destruction of subchondral bone, soft-tissue swelling, capsular calcifications, and intra-articular loose bodies.

A 45-year-old patient with a history of hyperparathyroidism presents with recurrent episodes of knee pain. X-rays reveal chondrocalcinosis of the medial and lateral menisci. The most likely diagnosis which would account for the patient's pain and x-ray findings is:

Pseudogout chondrocalcinosis = Pseudogout A definitive diagnosis of CPPD arthropathy (ie, pseudogout) requires the identification of CPPD crystals from joint fluid; however, the radiologic findings in this case are diagnostic. CPPD deposition disease can be associated with hyperparathyroidism, hemochromatosis, and amyloidosis. It is weakly associated with hypothyroidism. Chondrocalcinosis is not seen in osteoarthritis, monosodium urate crystal arthropathy (ie, gout), or Paget's disease.

Rheumatoid factor is found in rheumatoid arthritis and which of the following other diseases?

Psoriatic arthritis CREST syndrome Sjogren syndrome

Which of the following is a known sequela of rheumatoid arthritis? Ulnar deviation of the wrist; radial deviation of the fingers Radial deviation of the wrist; ulnar deviation of the fingers Ulnar deviation of the wrist; ulnar deviation of the fingers

Radial deviation of the wrist; ulnar deviation of the fingers In rheumatoid arthritis, one of the sequelae is a deformity resulting in radial wrist deviation with ulnar finger deviation

Which treatments may best prevent progression and improve symptoms of Complex Regional Pain Syndrome

Range of motion exercises and NSAIDs

Order of recovery in central cord syndrome

Recovery tends to start in the lower extremities then bladder function improvement is seen then proximal upper extremities improve and then finally intrinsic hand function improves

A 63-year-old patient with amyotrophic lateral sclerosis presents to your clinic with insidious onset of dyspnea over the last several months. What is the most likely abnormality on pulmonary function testing?

Reduced Tidal volume In restrictive lung disease, the primary abnormality is low tidal volume. In extrinsic restrictive lung disease (neuromuscular disease including amyotrophic lateral sclerosis, paralysis and kyphoscoliosis), this is due to weakness of the chest wall muscles. In intrinsic restrictive lung disease (pulmonary fibrosis, sarcoidosis), this is due to noncompliant lung tissue. In obstructive lung disease (asthma, COPD), abnormalities include increased residual volume and reduced forced expiratory volume due to incomplete exhalation or obstruction.

In the treatment of lymphedema, which condition is a relative contraindication to performing complete decongestive therapy?

Relative contraindications to complete decongestive therapy include significant congestive heart failure, acute deep venous thrombosis, acute or untreated infection or inflammation of the affected limb, and fracture. Lymphatic drainage massage should be avoided over concurrently irradiated soft tissues, but a history of previous radiation therapy is not a contraindication for therapy.

Which is true of Hereditary Inclusion Body Myopathy?

Relatively spares function of the quadriceps The myopathy described here is a hereditary Inclusion Body Myopathy (HIBM). Hereditary IBM is most commonly autosomal recessive, affects distal lower extremity muscles first, and spares the quadriceps even in advanced disease.

Which peripheral nerve stimulates the bladder to facilitate urine storage?

Remember, the hypogastric ("hippogastric") nerve makes your bladder as big as a hippo! Thus, it facilitates urine storage with all that extra space.

Diseases of which body system is the leading cause of death in the first month following spinal cord injury?

Respiratory! followed by cardiac, ID, cancer and PE

Which technical error can artificially increase (prolong) distal sensory latency when performing antidromic sensory nerve conduction studies?

Reversing the stimulator anode and cathode Depolarization occurs under the cathode with hyperpolarization beneath the anode. Placing the anode over the measured stimulus site and the cathode proximally results in depolarization more proximally, effectively increasing the distance between the stimulus and recording electrodes, and prolonging the distal sensory latency. An increase in the stimulus intensity or pulse width results in wider spread of the site of depolarization some distance from the cathode, effectively reducing the distance between the stimulating and recording electrodes, and shortening the distal sensory latency. The reference electrode should be placed more than 4 cm distal to the recording electrode; a shorter interelectrode distance has the effect of reducing SNAP amplitude and shortening the distal sensory latency due to differential amplitude effects of common mode signals being recorded.

A 52 year-old male who was previously active and healthy begins to notice muscle twitching and difficulty swallowing salivary secretions over the course of 2 months. He then begins to notice some weakness in his right arm and left leg. Electrodiagnostic studies reveal probable amyotrophic lateral sclerosis. He is started on riluzole by his neurologist. At his initial consultation with you, he asks you about the mechanism of riluzole. You respond with:

Riluzole blocks glutamate action in the central nervous system Amyotrophic lateral sclerosis is a progressive disease of upper and lower motor neurons. The median survival rate is 2-3 years after diagnosis, however the median survival rate is 1 year after diagnosis in those with bulbar symptoms. Treatment consists of physical therapy and submaximal exercise in addition to consideration of assisted ventilation and early nutritional intervention to supplement calories. Riluzole is an antiglutamate medication that has been shown to slow disease and prolong ventilator time.

WHat type of chart provides a graphic depiction of teh progrss of a quality improvement activity over time?

Run chart

What is the most common location of a pressure ulcer in a patient with spinal cord injury within the first 2 years?

Sacrum

list some causes of intrinsic/ interstitial lung diseases

Sarcoid IPF systemic sclerosis, asbestosis

Which part of the scaphoid bone has poor blood supply, rendering scaphoid fractures at that location susceptible to which feared complication?

Scaphoid fractures present following trauma/FOOSH (fall on outstretched hand) and typically cause pain in the anatomic snuffbox. X-rays/CT scan/MRI may reveal a fracture of the scaphoid; if this fracture occurs within the proximal ⅓ of the bone, orthopedic surgery consultation is required. If within the distal ⅔ of the bone, thumb spica cast is appropriate. If initial x-rays are negative for fracture, but you still suspect a scaphoid fracture, place thumb in thumb spica cast and repeat x-rays in 2 weeks, as the fracture may reveal itself at that time. You can also consider CT/MRI if x-rays are negative. Pain in the anatomic snuffbox is a scaphoid fracture until proven otherwise, even if initial x-rays are negative.

The conjoint hamstring tendon houses which tendons?

Semitendinosus, biceps femoris

A 30 year-old male develops left knee pain and swelling associated with fever. Synovial fluid analysis shows 130k WBCs with 97% neutrophils. What is the most likely etiology for this disease?

Septic arthritis in adults is typically due to N. gonorrhea infection. Synovial fluid analysis in septic arthritis reveals > 100k WBCs and overwhelming neutrophil (PMN) predominance. Septic arthritis in children is typically due to staph aureus.

In an inpatient rehabilitation setting, greater functional gains are typically seen in which individuals with which diagnosis? TBI ischemic stroke primary brain tumor metastatic brain tumor

Several studies have shown significant functional improvements from admission to discharge as measured by Functional Independence Measure (FIM) scores. Overall FIM change, FIM efficiency, and discharge to home rates were comparable between primary and metastatic tumor groups. Patients admitted to inpatient rehabilitation facilities with a diagnosis of traumatic brain injury tend to have greater total FIM change compared to patients with brain tumors. Patients admitted to inpatient rehabilitation facilities with a diagnosis of stroke appear to have equivocal total FIM change compared to patients with brain tumors. However, one study noted a greater FIM efficiency in the tumor cohorts. Researchers have found that patients who received chemotherapy or radiation during inpatient rehabilitation did not have a significant difference in length of stay (LOS) or FIM gains when compared to those who did not.

A 42 year old female presents with a cervical disk herniation at C 5-C6 on magnetic resonance imaging. SHe complains of radicular pain down her lateral left upper extremity to the thumb for 6 months as well as numbness of the elft thumb. No left upper extremity weakness is noted. Her EMG is normal and reveals normal nerve conduction of the left upper extremity of teh cervical paraspinals. What isthe most likley explanation for this patients normal nerve conduction and EMG studies?

She has a pure sensory lesion of the cervical nerve roots a pure sensory lesion of the nerve roots may be present in a normal EMG study

Which of the following is commonly used as an outcome measure to assess perception of physical and emotional functioning in patients with chronic pain?

Short Form 36 The SF-36® and the shorter SF-12® is a measure of quality of life with questions covering multiple domains. SF-36 Domains include "physical functioning; role limitations due to physical health; role limitations due to emotional problems; energy/fatigue; emotional well-being; social functioning; pain; general health". These questionnaires are owned and marketed by Medical Outcomes Trust. They have a considerable research base and assess a broad range of activities of daily living relevant to disability.

A 4-year-old child suffers an ankle inversion. When you examine him the following day, he has mild swelling laterally and is tender to palpation approximately 2-3 cm proximal to the distal aspect of the lateral malleolus and he reacts to pinch. The remainder of the examination is limited due to pain. Radiographs appear normal. What is the management of this patient?

Short leg non-weight bearing cast for 3 weeks Given the mechanism of injury, location of pain, and "normal" radiographs, the boy likely has a Salter Harris I Fracture of the distal fibular physis. A Salter Harris I fracture is a transverse fracture through the hypertrophic zone of the physis, and is typically not visible on radiographs though there may be widening of the growth plate. A walking boot is not sufficient to immobilize the ankle joint in a toddler. A CT scan is not required given there is not any significant abnormality on radiographs. A MRI to rule out an ATFL tear is not indicated. There is a significantly greater likelihood of a bony versus ligamentous injury.

Which of the following imaging findings is most consistent with congenital cervical stenosis?

Shortened pedicles

Which of the following lab results would you expect to see in classic Sjogren syndrome? ANA: antinuclear antibody. RF: rheumatoid factor.

Sjogren syndrome classically is ANA+ and RF+.

A 14 year-old male with a history of obesity presents with sudden-onset left hip pain. He denies trauma. He has been healthy other than a diarrheal illness 2 weeks ago. Hip x-rays demonstrate a widened growth plate and femoral head sliding off the growth plate by approximately 40%. Which of the following grades could correctly be assigned to these x-ray findings?

Slipped capital femoral epiphysis (SCFE) involves sudden-onset hip/groin pain due to the femoral head sliding off the growth plate, typically in obese adolescent males. X-ray grading is defined by the percentage of slippage of the femoral head off the growth plate. Grade 1: 0-33% sliding. Grade 2: 34-50% sliding. Grade 3: >50% sliding. There is no grade 4. Treatment is orthopedic surgery.

Which of the following is a recommended static alignment for an uncomplicated transtibial amputee with no significant knee contracture?

Socket positioned in 5° to 10° of anterior tilt (flexion) The socket is prepositioned in 5° of adduction and 5° to 10° of anterior tilt. The anterior tilt allows loading of the soft tissues of the anterior surface of the residual limb. The foot is slightly inset relative to the socket. The patella tendon bar is located halfway between the tibial tubercle and the distal end of the patella. The posterior brim of the socket should end about an inch below the patella tendon bar, with relief made for the hamstring tendons.

what are the 6 determinants of gait

The 6 determinants of gait are as follows: pelvic tilt, pelvic rotation, pelvic lateral excursion, foot mechanisms, knee mechanisms, knee flexion.

The most common risk factor for congenital limb deficiency is:

Vascular Condition Congenital limb deficiencies occur during the 3rd to 8th weeks of gestation. It is estimated that 34% are vascular, 32% have no known cause, 30% are genetic, and 4% are caused by a teratogenic agent.

With aerobic training, a patient can alter the ratio of which of the following?

With aerobic training, a patient can increase their ratio of type IIa to type IIb muscle fibers (increasing the proportion of type IIa fibers relative to type IIb). Type I muscle fiber amount remains roughly the same.

Which centrally acting sympatholytic hypertensive medication can worsen erectile dysfunction?

clonidine

Other than rest, ice and NSAIDS, what can be done for quadriceps tendonitis?

cryotherapy to reduce secondary hypoxic injury

Considering safety and comfort, what is the preferred location for needle electrode insertion when studying the diaphragm?

The eighth or ninth interspace, at the anterior axillary line

hormonal changes in the older adult play an important part underlying the etiology of frailty. Such abnormalities include increase in what labs and decrease in what labs?

decreased levels of DHEA-S, sex steroids, 25 OH vitamin D, Growth hormone increase in cortisol

A 67 year-old female with a history of facial trauma presents with complaints of crying every time she eats. She denies any actual emotional disturbances. What is the most likely etiology of this problem?

Synkinesis is the inappropriate regeneration of the facial nerve after it is damaged, leading to the nerve reinnervating muscles inappropriately in such a fashion that the patient may end up doing two "facial nerve" activities at once when the patient only means to do one of these. For example, blinking and moving their mouth at the same time, or salivating whenever they cry.

supplemental oxygen in patients with chronic obstructive pulmonary disease has been shown to?

improve walking tolerance

Which of the following is the most common form of spina bifida?

Spina bifida occulta is the most common form, followed by myelomeningocele.

A patient diagnosed with a lateral medullary stroke exhibits ipsilateral loss of pain/ temperature sensation of the face. Where is the corresponding neuroanatomic site?

Spinal tract of the trigeminal nerve

how are electrophysiological tests helpful in the acute period after a SCI

Somatosensory Evoked potentials, SSEP, can be performed before spinal shock has resolved.so it can help determine if an uncooperative patient has a SCI

A patient presents with pain in his ring finger after an opponent tried to slap a basketball out of his hand. On exam flexion and extension of his DIP is intact. X-ray of his finger shows a non-displaced distal phalangeal fracture. What is the next step in his treatment?

Splint immobilizing DIP joint only The goal of treatment for any finger injury is to restore the normal function of the finger. Restoration of bony anatomy is the basis for returning normal function; however, an anatomic reduction is not always necessary to achieve this goal, especially if it comes at the cost of soft tissue scarring and loss of motion. Early motion prevents adhesions of the gliding soft tissues of the extensor and flexor tendon systems and prevents contracture of the joint capsules. Immobilization of fingers much beyond 4 weeks will lead to long-term stiffness due to extensor tendon and joint capsular scarring. For example, non-articular phalangeal fractures treated with closed reduction and splinting are mobilized after 3-4 weeks, once the fractured phalanx is less tender. Even if splinting of one joint is needed, splints should be made small enough to allow early motion of uninjured joints. Therefore, in a case of a non-displaced distal phalanx fracture splinting that immobilizes the DIP joint only is sufficient. Closed non-displaced or minimally displaced fractures with acceptable alignment that are the result of a low-energy trauma usually have sufficient supporting tissues remaining intact making them stable and amenable to treatment by protected mobilization, either with local splinting of the fracture or buddy taping to adjacent fingers. Fractures with rotational or angular misalignment may be amenable to closed reduction and splinting, but these fractures are at risk for incomplete reduction and recurrent deformity. These more unstable fractures require careful and frequent clinical and radiographic follow-up. Surgical treatment is indicated for any fractures of the articular surface, open fractures, fractures with significant shortening or malrotation, and fractures which fail closed reduction. Delayed treatment of these surgically indicated fractures is always more difficult, with worse functional outcomes due to stiffness, incomplete deformity correction, and post-traumatic arthritis.

A 61 year-old male presents with low back pain radiating down the bilateral lower limbs with numbness and tingling in the legs. MRI of the lumbar spine demonstrates 55% slippage of the L3 vertebral body on top of the L4 vertebral body. X-rays reveal no dynamic instability. On exam he has intact strength, sensation, and reflexes. What is the next best step?

Spondylolisthesis is the slippage of one vertebral body over the top of another. Grade 1 is 1-25% displacement when comparing the two vertebral body surfaces. Grade 2 is 26-50% slippage; grade 3 is 51-75% slippage; grade 4 is 76-100% slippage; grade 5 is beyond 100%. Grades 1 and 2 can be managed with PT. Grade 3 if asymptomatic (other than low back pain) can be managed with PT. Grade 3 with neurologic symptoms should have a surgical consultation. Grades 4 and 5 require surgical consult.

Which stage of complex regional pain syndrome (CRPS 1) is characterized by burning pain, dependent edema, and vasomotor instability?

Stage 1, the acute phase, is characterized by burning pain, dependent edema, redness, hyperhidrosis, coolness to touch. Stage 2, subacute phase, is characterized by severe pain, fixed edema, and cyanosis or pallor. Stage 3, chronic phase, pain may have subsided, edema subsided, and the extremity is generally stiff. There is no defined 4th stage. The three clinical stages of type 1 complex regional pain syndrome (CRPS 1) are acute, subacute, and chronic. The acute form lasts approximately 3 months. Pain, often burning in nature, is one of the first symptoms that initially limits function. Swelling, redness with vasomotor instability that worsens with dependency, hyperhidrosis, and coolness to the touch are common physical find-ings. Demineralization of the underlying bony skeleton begins because of disuse. If the process is not arrested or reversed in the acute phase, the condition may progress to the subacute stage, which can last for up to 9 months. The patient develops persistent severe pain in the extremity and fixed edema that would have been reversible with elevation during the acute phase. The redness of the acute stage gives way to cyanosis or pallor and hyperhidrosis to dry skin. Loss of function progresses, both because of increased pain and fibrosis of the joints caused by chronic inflammation. In the hand, this leads to flexion deformity of the fingers. The skin and subcutaneous tissues begin to atrophy. Demineralization of the underlying bony skeleton be-comes pronounced. If the process continues, the chronic phase may develop approximately 1 year after disease onset. This stage may last for many years or can be permanent. Pain is more variable during this period. It may continue undiminished or abate. Edema tends to subside over time, leaving fibrosis around the involved joints. The skin is dry, pale, cool, and shiny. Flexion and extension creases are absent. Loss of function and stiffness are marked, and osteoporosis is extreme. In the upper extremity, this can manifest as a frozen shoulder and claw hand.

Bilateral radiculopathy of which spinal level will theoretically not cause impaired storage of urine or impaired bladder contraction? S2 L4 L2 T12

Storage of urine is controlled by the pudendal nerve innervation of the external urethral sphincter via S2-S4 nerve roots, and the hypogastric nerve's innervation of the internal urethral sphincter via T11-L2. The hypogastric nerve also innervates the bladder wall promoting relaxation. For bladder contraction and voiding, the pelvic splanchnic nerve (S2-S4) causes detrusor contraction and relaxation of the internal urethral sphincter. In the above question, only L4 is a nerve root not involved in the innervation of the detrusor muscle or either the internal or external urethral sphincter.

The Americans with Disabilities Act protects individuals from employment discrimination if they have a physical or mental impairment that:

Substantially limits a major life activity Employment discrimination is prohibited against qualified individuals with disabilities. An individual is considered to have a disability if she or he has a physical or mental impairment that substantially limits one or more major life activities such as seeing, hearing, speaking, walking, breathing, performing manual tasks, learning, caring for oneself, and working. A qualified individual with a disability is a person who meets legitimate skill, experience, education, or other requirements of an employment position that she or he holds or seeks, and who can perform the essential functions of the position with or without reasonable accommodation. Reasonable accommodation is any modification or adjustment to a job or the work environment that will enable a qualified applicant or employee with a disability to participate in the application process or to perform essential job functions. Employers are not required to lower quality or quantity standards as an accommodation; nor are they obligated to provide personal use items such as glasses or hearing aids.

Which blood vessel is most likely to compress the trigeminal nerve?

Superior Cerebellar artery The most often cited vascular structures associated with trigeminal neuralgia (in order) include the superior cerebellar artery, the anterior inferior cerebellar artery, the posterior inferior cerebellar artery, and the superior petrosal vein.

A physiatrist is using ultrasound to examine a patient's shoulder. What is the anterior boundary of the rotator interval?

Superior aspect of subscapularis tendon Improper identification of the rotator interval may lead to false diagnosis. The rotator interval is bordered anteriorly by the superior aspect of the subscapularis tendon and posteriorly by the anterior aspect of supraspinatous tendon. Contents of the rotator interval include the capsule, superior glenohumeral ligament, coracohumeral ligament, and long head of the biceps tendon. The rotator interval is best viewed with the patient in modified Crass position which allows for inspection of the intracapsular biceps tendon. The biceps tendon will appear hyperechoic on ultrasound with a hypoechoic separation from the supraspinatus.

Describe the path of the F wave

Supramaximal stimulation ,an antidromic motor response to anterior horn cells which produces an oprthodromic motor response that travels back to the recording electrode.

Which of the following nerves is associated with the spinoglenoid notch? Axillary nerve Dorsal scapular nerve\ Suprascapular nerve

Suprascapular nerve The suprascapular nerve innervates the supraspinatus, then passes through the spinoglenoid notch to innervate the infraspinatus. This is an important anatomical consideration when performing an EMG involving both supraspinatus and infraspinatus. The axillary nerve passes through the quadrilateral space of the axilla.

A 6-month-old boy with right Erb's palsy is not able to bring his right hand to his mouth and has weak wrist extension. His right upper extremity passive range of motion is within normal limits. Which of the following is the most appropriate next step in treatment?

Surgical referral This patient demonstrates weakness in elbow flexion noted by inability to reach the mouth. Surgical intervention is commonly recommended for those having less-than-antigravity strength in elbow flexion by 6 months of age. Patients with Erb's palsy who had surgery at 6 months did better than those who spontaneously recovered elbow flexion at 5 months.

a 16 year old male is seen by hjis physicina for a routine eval prior to participating in sports. Which of the following parameters is most likely to decrease during excercise? - inotropy -chronotropy -stroke volume -systemic vascular resistance -venous return

Systemic vascular resistance Exercise causes increased cardaic output, stroke volume, chronotropic, inotropy, and venous return. THeswe are accompanied by decreased systemic vascular resistance

You ar evaluating a patient with UE CRPS she has had only partial releif with stellate ganglion blockade. You explain that the Kuntz nerves may supply part of teh sympathetic nerve innervation to her arm. The Kuntz nerves are a contribution from ???

T 2 sympathetic fibers to the upper extremity

TAR syndrome is associated with what problem?

TAR = thrombocytopenia with absence of radius syndrome. associated with thrombocyopenia. X ray shows absent radius.

Which class of antidepressant medication has a narrow therapeutic index and is associated with fatal heart block or ventricular arrythmias at toxic doses?

TCA

HOw do steroids affect the growth velocity in children?

THe growth effects of glucocorticosteroids are related to teh type, dose and duration of teh exposure

What does an ulnar claw hand look like?

THe hand will show hyper extension of the MCP and flexion of the distal and proximal interphalangeal joints of the 4th and 5 th digits. There is also loss of abfduction and adduction of the digits due to paralysis of palmer and dorsal interossei and medial 2 lumbricals.

Which of the followign is true about the changes in incidence rate of acute traumatic spinal cord injury in the United States between 1993 and 2012?

THe total number of cases increased but hte incidence rate remained relatively stable. Due to increse in population.

which meds can raise the serum urate c oncentrations and thus increase teh risk for gout? which bP med reduces the risk?

THiazide diuretics, loop diuretics Losartan

A patient who had a right MCA stroke presented one hour after symptom onset and received intravenous thrombolysis with t-PA, improving his NIHSS from a 12 to a 5. He is ambulating with physical therapy on the acute medical floor. Which factor has greatest evidence to predict his chances of going home?

TPA within the first 3 hours Patients who receive intravenous thrombolysis for acute ischemic stroke are more likely to be discharged directly home after hospitalization with outpatient neurorehabilitation therapies. There is no correlation with the exact time within the three-hour window. His premorbid functional status, location of stroke, and early mobilization are important components of his rehabilitation; however, they have no bearing on the efficacy of t-PA therapy.

What is the greatest predictor ofd succesful amputation wound healing?

TPO transcutaneous partial pressure oxygen

Fracture of which of the following parts of the talus places the patient at highest risk of avascular necrosis (AVN)?

Talar body (AKA talar dome) fractures carry the highest risk of AVN over other types of talus fractures; thus, they require orthopedic surgery referral.

What test is the best method for determining level of amputation?

TcPO2 The transcuatneous oxygen pressure is used to find the level of amputation with enough blood perfusion to increase chance of succesful wound healing and decrease risk for re amputation. Based on clinical research studies, a median TcPO2 of 28 mmHg is likely to provide sufficient

What is the anatomic location of a stroke which produces impaired expression, comprehension, and repetition, with normal fluency?

Temporal gyrus

While on inpatient rounds on the stroke unit of the rehab hospital, your attending discusses the various rehabilitation methods. Which method focuses on normalization of tone and inhibition of primitive patterns of movement?

The Bobath approach is described in the question stem, with a focus on normalization of tone and inhibition of primitive patterns of movement at its core principles. The Brunnstromm Approach uses primitive synergistic patterns to attempt to improve motor control. The Rood Approach attempts to modify muscle tone and voluntary motor activity using cutaneous sensorimotor stimulation. Lastly, the proprioceptive neuromuscular facilitation uses spiral and diagonal movements with the goal of facilitating more functional movements rather than attempting to strengthen individual muscle groups.

In the Brunnstrom stages of recovery from hemiparetic stroke, at what stage does the patient develop peak spasticity?

The Brunnstrom hemiparetic stroke recovery stages are as follows: ○1 = totally flaccid ○2 = spasticity, hyperreflexia, UE flexor synergy pattern, LE extensor synergy pattern ○3 = spasticity peaks; control over synergy patterns begins ○4 = spasticity decreases; control is maximized ○5 = complex voluntary movements ○6 = spasticity gone ○7 = normal

The extensor digitorum attaches to which of the following structures? Distal phalanx Proximal phalanx Both distal and proximal phalanges

The ED muscle attaches to a thick piece of tissue called the extensor expansion, which pulls on both the DIPs and PIPs, causing extension of both.

Which of the following muscles is innervated by the listed neural pathway? S1, S2; sciatic nerve; tibial nerve; medial plantar nerve. Abductor digiti quinti pedis (ADQP) Flexor hallucis brevis (FHB)

The FHB is innervated by this pathway, as well as the abductor hallucis brevis (AHB). The FHL and FDL are tibial nerve-innervated. The ADQP is lateral plantar nerve-innervated.

Which of the following receptors, when stimulated, increases presynaptic chloride influx into the neuron?

The GABA-A receptor causes presynaptic chloride influx into the axon. The GABA-B1 receptor inhibits presynaptic calcium influx into the neuron. GABA-B2 increases postsynaptic potassium conductance out of the neuron. All of these effects serve to reduce synaptic transmission of action potentials. Tizanidine binds to alpha-2 receptors to stimulate them, which inhibits the spinal reflex arc.

The Gate Control Theory of pain modulation functions due to which of the following anatomical principles?

The Gate Control Theory is based on the anatomic principle that large-diameter light touch fibers transmit their impulses onto the same neurons in the spinal cord's substantia gelatinosa as do smaller-diameter pain fibers, thus allowing light touch impulses to inhibit the transmission of pain impulses up to the brain by "taking over the sensation transmission highway".

List the HLA B 27 spondyloarthropathies

The HLA-B27 spondyloarthropathies include psoriatic arthritis, ankylosing spondylitis, oligoarticular JIA, enteropathic arthropathy, and reactive arthritis (formerly known as Reiter syndrome).

A 41 year-old male attempts to squat down and lift a car by reaching his hands under the rear bumper and lifting upward. He feels sudden onset anterior left elbow pain. You suspect a distal biceps tendon tear and perform the Hook test. All of the following are possible reasons for a false negative Hook test except which choice?

The Hook test involves placing the patient's elbow in 90 degrees of flexion and using the examiner's finger to try to "hook" the distal biceps tendon out of the elbow; a negative test indicates a firm, intact biceps tendon which is properly attached to the radial tuberosity; a positive test reveals a floppy "wet noodle" biceps tendon, indicating distal biceps tendon complete tear. All of the following answer choices could lead to a negative Hook test that is falsely negative, except palpating an intact brachioradialis tendon, which would be distal to the elbow and not within the elbow crease; the brachioradialis muscle is also lateral to where the biceps tendon would be palpated via the Hook test. By performing the Hook test in someone with a biceps tendon tear, and feeling a firm structure that is not floppy, the examiner must maintain vigilance and make sure that he or she is not accidentally palpating the brachialis tendon, intact bicipital aponeurosis, or palpating a partial tear of the biceps tendon.

A patient experiences a sudden-onset severe headache with nausea and vomiting while performing a heavy deadlift. He is rushed to the ED where the ED physician documents the patient with headache, neck stiffness, mild left hemiparesis, and confusion. What is the most appropriate Hunt and Hess score for this patient?

The Hunt and Hess scale for subarachnoid hemorrhages can be simplified as follows: Grade 1: roughly asymptomatic (potentially mild symptoms) with no neurologic deficits. Grade 2: Severe headache, neck stiffness are present with no major neurologic deficit or confusion. Grade 3: Headache, neck stiffness, confusion, with focal neurologic deficit. Grade 4: Extremely confused (think heavily inebriated), headache, neck stiffness, with severe focal neurologic deficit. Grade 5: coma.

What are the grades in the Hunt and Hess Scale?

The Hunt and Hess scale for subarachnoid hemorrhages can be simplified as follows: Grade 1: roughly asymptomatic (potentially mild symptoms) with no neurologic deficits. Grade 2: Severe headache, neck stiffness are present with no major neurologic deficit or confusion. Grade 3: Headache, neck stiffness, confusion, with focal neurologic deficit. Grade 4: Extremely confused (think heavily inebriated), headache, neck stiffness, with severe focal neurologic deficit. Grade 5: coma.

The lateral antebrachial cutaneous nerve (LAC) arises off of what part of the brachial plexus?

The LAC is actually just the terminal portion of the musculocutaneous nerve, after it has innervated all its muscles.

You are performing a physical exam on a patient with knee pain. You flex the hip and knee, externally rotate the ankle, and apply a valgus force to the knee while extending the knee. Which structure are you assessing for a tear?

The McMurray test is described here. It is a test for meniscus tears in the knee. By externally rotating the ankle and applying a valgus force while extending the knee, you are testing the medial meniscus for tears; internal rotation with varus force is a test for the lateral meniscus. Both are called the McMurray test. Tip: the heel itself moves toward the meniscus being tested; i.e. with external rotation, the heel moves medially, and thus, the medial meniscus is tested (with external rotation + valgus stress to the knee).

The National Pressure Ulcer Advisory Panel (NPUAP) pressure injury staging system? 1, 2, 3, 4

The National Pressure Ulcer Advisory Panel (NPUAP) pressure injury staging system is as follows: ●1 = nonblanchable erythema (red skin that stays red when you press on it) ●2 = skin breakdown extends into the dermis ●3 = through the dermis with subQ fat exposed ●4 = muscle, tendon, or bone is exposed ●Deep tissue injury = purple, intact skin, but the wound bed cannot be visualized ●Unstageable = really a stage 3 or 4, but you can't tell because there is so much slough/debris

Some individuals possess a normal anatomic variant by which their median nerve motor fibers in the hand cross over to join the ulnar nerve. Which of the following is the correct name for this variant?

The Riche-Cannieu anastomosis involves median motor fibers in the hand crossing over to join the ulnar nerve; classically this can cause the ulnar nerve to supply motor control to the entire hand. The Martin-Gruber anastomosis is another anatomic variant involving median nerve fibers crossing over in the forearm to innervate the ulnar nerve and supply some ulnar nerve muscles in the hand. "Kertley-Thompson anastomosis" and "Shelley-Webster anastomosis" are fictional. Perhaps Kertley and Thompson are indeed out there, searching for their anastomosis of fame.

The blink reflex study tests CN V and CN VII in the brainstem and peripherally. CN V has two nuclei being tested (Vm and Vs). The Vm nucleus lies where. Vs nucleus?

The Vm nucleus lies within the pons. It accepts CN V input from the face and conducts it to the ipsilateral CN VII nucleus, causing an ipsilateral blink (R1 response). It also simultaneously conducts the impulse to the medulla where the Vs nucleus lies. Vs accepts the impulse from Vm and sends the impulse to the bilateral CN VII nuclei, which then causes a bilateral blink via the orbicularis oculi muscles (R2 response).

The area of hypovascularity of the achilles tendon (leading to tears in this region) is generally at which of the following locations on the tendon? The musculotendonous junction Distal 2-6 cm of the tendon

The achilles tendon's zone of hypovascularity is generally within the distal 2-6 cm of the tendon, predisposing this region to tendon tears due to the poor blood supply.

The purpose of the muscles within the lateral compartment of the lower leg is to perform which of the following actions?

The fibularis longus and brevis are located within the lower leg's lateral compartment along with the superficial fibular nerve which innervates these muscles. Their action is to cause plantarflexion and eversion of the ankle.

You are seeing a 26 year-old marathon runner in follow-up. You previously prescribed physical therapy for achilles tendon pain. Since finishing the physical therapy, the patient notes minimal relief of their pain. You decide to perform a diagnostic ultrasound scan of the achilles tendon. Immediately you notice a thickened, hypoechoic appearance of the tendon. Color doppler reveals punctate pulsations of blood flow within the tendon. What is the etiology of this condition, defined by the totality of these ultrasound findings?

The findings above describe findings of tendonosis, which is a chronic degenerative process. This is typically due to poor strength and overuse of the tendon. Subsequent neurogenic inflammation can develop, as indicated by the color doppler flow. Physical therapy is a good starting treatment for this. Failing this, it is reasonable to try regenerative tendon interventions with ultrasound guidance.

The function of the anterior talofibular ligament (ATFL) is to do what?

The function of the ATFL is to resist anterior talar translation. The ATFL's integrity can be tested using the anterior drawer test of the ankle, in which the examiner stabilizes the calf distally and pulls anteriorly on the calcaneus, thus drawing the talus anteriorly, separating it away from the fibula and, thus, stretching/stressing the ATFL. If it is torn, there will be no firm endpoint. << Previous

50-year-old patient reports a headache that begins in the back of the head, radiating to the vertex of the scalp. The patient denies any other associated symptoms, including fever, neck pain, lacrimation, nausea, visual disturbance, or sensorimotor deficits. On exam, they have pain with palpation just lateral to the occipital protuberance. What is the most likely diagnosis for this case?

The greater occipital nerve is the medial branch of the dorsal ramus of the second cervical nerve (C2). It provides sensory innervation to the posterior scalp up to the vertex, among other areas. It is located lateral to the external occipital protuberance, just medial to the occipital artery, where is can be blocked with injection of local anesthetic. Tension headache is usually described as a band-like pain around the head. Migraine headache is generally described as a unilateral, severe headache, often associated with nausea, photosensitivity, and aggravated by physical activity. Cluster headache is severe unilateral orbital/supraorbital pain with may be associated with lacrimation, ptosis and nasal congestion

Which orthoses would be most appropriate in the setting of a type 2 dens fracture?

The halo vest and Minerva jacket are the appropriate orthoses for use in unstable cervical spine fractures, such as a type 2 dens fracture. The philadelphia collar is useful for stable cervical spine fractures. The Jewett brace is a thoracolumbosacral orthosis (TLSO) and not useful for cervical spine fractures.

What are the borders of guyon's canal?

The hook of the Hamate forms the lateral (radial) border of Guyon's canal, while the pisiform forms the medial border. The hypothenar muscles form the floor of Guyon's canal along with the flexor retinaculum. The ulnar nerve can become entrapped within Guyon's canal, producing an ulnar neuropathy at the wrist. A hook of the Hamate fracture can cause ulnar neuropathy at the wrist.

What is the function of the iliofemoral ligament regarding the hip?

The iliofemoral ligament extends from the ileum to femur on the anterior side of the hip joint. Its function is to limit hip abduction, extension, and external rotation.

The final ligament pierced by a spinal needle during an epidural injection is?

The ligamentum flavum is the final ligament that is pierced by an epidural needle during an epidural injection. Once piercing the ligamentum flavum, classically a "loss of resistance" is felt in the syringe, meaning that it is now much easier to inject medication through the syringe, now that the needle has entered the relatively "wide open" epidural space.

The most common cause of death in acute spinal cord injury cases is:

The most common cause of death in acute SCI is pulmonary embolism. The most common cause of death in chronic SCI is pneumonia.

What is the most common cause of mortality after total hip replacement?

The most common cause of mortality after total hip replacement is cardiovascular complications followed by thromboembolic events.

What is the most common cause of traumatic brain injury in pediatric patients?

The most common cause of overall TBIs is MVAs. The most common cause in elderly patients is falls. The most common cause in children is MVAs

In SCI patients, the most common time to onset of heterotopic ossification following injury is:

The most common onset is 1-4 months, but can be present up to the first 6 months.

Which of the following is the most common type of spinal cord injury?

The most common type of SCI is incomplete tetraplegia, followed by incomplete paraplegia, complete paraplegia, and complete tetraplegia, in that descending order, as of the most recent statistics.

The Bobath (neurodevelopmental) technique of rehabilitating a stroke patient aims to do which of the following?

The neurodevelopmental approach to rehabilitation seeks to eliminate abnormal spasticity, flexor synergy, and primitive reflexes. The Brunnstrom approach involves promoting these factors to facilitate function in ambulation and ADLs. The Rood approach involves sensory stimulation to help retrain proper movement patterns. Proprioceptive neuromuscular facilitation (PNF) involves performing diagonal movement patterns to facilitate a more natural movement for the limbs.

67-year-old male with a history of cervical spondylosis presents with right hand paresthesias. His internist orders an EMG/NCS of his right upper extremity, which reveals absent median nerve SNAP (sensory nerve action potential) and positive sharp waves in the APB (abductor pollicis brevis) muscle during the needle exam. He undergoes surgical treatment. Following surgery, which of the following orthoses is most appropriate?

The patient has right hand paresthesias in the setting of severe median neuropathy findings on EMG (absent median nerve SNAP and active denervation of APB (PSWs)) which indicates median neuropathy as the likely diagnosis. The most appropriate surgical intervention would be carpal tunnel release given the severity of the disease based on electrodiagnostic findings. Following carpal tunnel release, a gel shell splint is used; this is a nonarticular brace (doesn't cross joint meaning it doesn't restrict ROM) used to help prevent hypertrophy of the surgical scar by maintaining pressure against the healing incision. A thumb spica splint limits thumb ROM, and can be used for DeQuervain tenosynovitis, skier's thumb, dis

All of the following bones belong in the distal row of carpal bones except which of the following?

The proximal row of carpal bones contains scaphoid, lunate, triquetrum, and pisiform. The distal row of carpal bones contains trapezium, trapezoid, capitate, hamate.

The proximal row of carpal bones contains ?. The distal row of carpal bones contains?.

The proximal row of carpal bones contains scaphoid, lunate, triquetrum, and pisiform. The distal row of carpal bones contains trapezium, trapezoid, capitate, hamate.

Approximately 2 months after an ischemic stroke while in acute rehab, a patient develops shoulder pain on his hemiparetic side. He also notices some swelling in his hand, allodynia in the entire upper extremity, and increased sweating even at rest in the affected extremity. He is noted to have an IV in the same extremity. What is the likely cause of this patient's symptoms.

The question stem gives a history and physical signs concerning for CRPS type I, formerly known as reflex sympathetic dystrophy. It is often characterized by sympathetic-mediated pain, sensory abnormalities, allodynia, abnormal blood flow, and trophic changes. If an identifiable peripheral nerve injury occurs preceding signs/symptoms, CRPS type II also known as causalgia is the likely culprit. Superficial thrombophlebitis and acute DVT, although reasonable to put on a differential diagnosis for the above case, are ultimately meant to be distractors in this question.

During a blink reflex study, when stimulating the left trigeminal nerve, you detect a single left eye blink response. This is known as which of the following?

The quick, ipsilateral blink response is called the R1 response, and we record it from the orbicularis oculi muscle ipsilaterally to the side of stimulation.

The "roof" of the tarsal tunnel is also known as which of the following?

The roof of the tarsal tunnel can be thought of as the flexor retinaculum, under which the structures of the tarsal tunnel pass.

Which of the following is the primary structure that maintains the humeral head within the glenoid fossa?

The rotator cuff is chiefly what maintains proper contact of the humeral head within the glenoid fossa. When these muscles are weak (e.g. stroke) the humeral head is far more likely to sublux out of the glenoid fossa. The other structures listed are secondary to the rotator cuff in this role.

The strenuousness of an activity relative to basal metabolic rate is defined as which of the following?

The strenuousness of an activity relative to basal metabolic rate is defined as how many metabolic equivalents (METs) the activity is above basal metabolic rate. For example, lying flat is 1 MET, and walking on a treadmill is 3 METs (roughly).

You are reviewing left lower limb EMG/NCS findings and trying to propose a possible treatment plan to the patient. The patient is experiencing foot pain not relieved by NSAIDs or heat/ice. NCS reveals normal superficial fibular and sural SNAPs (sensory nerve action potentials), prolonged latency of medial and lateral plantar nerve SNAPs, and abnormal CMAP (compound muscle action potential) to abductor hallucis (AH). The CMAP to the extensor digitorum brevis (EDB) is normal. Needle EMG reveals decreased recruitment in AH, abductor digiti quinti pedis (ADQP), and lumbricals. EMG of tibialis anterior, EDB, gastrocnemius, fibularis longus, hamstrings, rectus femoris, tensor fascia lata, and lumbosacral paraspinals is normal. Which of the following is the most reasonable treatment strategy for this condition?

These findings suggest tarsal tunnel syndrome, a rare compression of the tibial nerve as it passes through the tarsal tunnel around the medial malleolus, underneath the flexor retinaculum. The flexor retinaculum is a common culprit that compresses the tibial nerve too tightly, causing the patient's symptoms of plantar foot pain with numbness/tingling and possible foot muscle weakness. Surgical release of the flexor retinaculum (akin to carpal tunnel release) is the best option of these choices (a structural solution for a structural problem). US-guided tarsal tunnel injection could also be considered. Bracing would not do anything to decompress the tarsal tunnel. Gabapentin also would not decompress the tarsal tunnel, but mask the symptoms while the nerve compression continues, potentially destroying axons over time. Peripheral nerve stimulation is not appropriate for this reason also.

A 21 year-old female complains of gradual onset worsening right hip pain. She is currently training for a marathon. She denies weakness, numbness, and tingling. X-rays reveal a cortical lucency along the inferior/medial femoral neck. There is pain during normal ambulation. What is the most appropriate next step in management?

This patient has developed a compression-side proximal femur stress factor. Risk factors include low BMI, poor nutrition, amenorrhea, osteoporosis. Compression-side fractures (on inferior/medial side of femoral neck) are more common than tension-side fractures (fractures along superior/lateral femoral neck) and can be treated with non-weight-bearing for a period of time, then gradual weight-bearing as tolerated, then gradually increase activity. Tension-side fractures are less stable and require orthopedic surgery consult for ORIF.

A 65 year-old female is brought to the ED due to sudden-onset right hemiplegia. On exam you discover she has impaired sensation to light touch on her right side. Her extraocular motility and visual acuity are normal bilaterally. When asked to stick out her tongue, her tongue deviates toward the left side. What is the most likely diagnosis?

This patient has suffered a stroke of her left medial medulla. In the medial medulla lies the corticospinal tract, medial lemniscus, and CN 12 nucleus. Thus, damage to these structures will cause, respectively, contralateral hemiparesis/plegia, contralateral impaired light touch, and ipsilateral CN 12 palsy (tongue deviates toward the side of the lesion). Weber syndrome presents with contralateral hemiparesis with ipsilateral CN 3 palsy.

A 27-year-old manual laborer presents with a 6-month history of right shoulder pain. He has a past medical history of a traumatic right shoulder dislocation 3 years ago. On physical exam, he has normal strength and sensation with symmetric reflexes. The shoulder apprehension test is positive. Impingement tests and the O'Brien active compression test are negative. What type of labral tear is most consistent with this presentation?

This patient presents with an anterior-inferior labrum tear related to chronic unidirectional anterior shoulder instability following a prior traumatic event (dislocation). This type of instability is common after a traumatic event as opposed to multidirectional instability, which is associated with congenital laxity or chronic repetitive microtrauma. Anatomically, unidirectional anterior instability involves disruption in the anterior-inferior glenohumeral joint capsule and anterior-inferior labrum with or without associated bony injury to the glenoid rim (Bankhart lesion) and humeral head (Hill-Sachs deformity). Superior labral anterior to posterior (SLAP) lesions may occur, but that possibility is less likely in this patient, because of his history and a negative O'Brien compression test.

55 year-old female presents to your clinic with the complaint of fever and elbow problems. She notes that for the past several weeks she has noticed her right elbow is "bigger" than the left elbow. She is a student and has been taking lots of college classes lately. She uses an elbow pad to no benefit. On exam, strength and sensation are intact. You notice that at the posterior elbow there is a very large swelling. It is compressible and warm. The patient notes some discomfort there and asks you to "do something". What is the next best step?

This patient presents with classic findings of olecranon bursitis, which is usually due to repetitive forces to the olecranon, such as grinding your elbow on a desk for several hours daily (art, desk work, etc.). A large pouch of fluid develops at the olecranon bursa because there is very little skin there to prevent it from expanding if it is inflamed. Treatment involves RICE, elbow pad, and considering aspiration. It is not usually an infectious process, but if the patient has a fever, then consider aspiration, culture, and antibiotics. This patient has failed conservative treatment, and aspiration of this bursa carries little risk if done properly. Caution the patient that there is a good chance it will recur following aspiration unless the irritant forces are removed that are causing the bursopathy in the first place.

A 47 year-old male presents with 3 weeks of nonpainful left elbow swelling. He denies numbness, tingling, weakness, fever, chills, or trauma. He is an architect and draws blueprints for buildings as part of his work. He is bothered by the cosmetic appearance of the swelling. On exam, you note a posterior elbow fluctuant mass which is nontender to palpation; there is no rash. Neurologic examination is normal in the upper extremities. You trial an elbow pad; he returns 1 month later stating that the swelling is unchanged. Which of the following is the next best step in management?

This patient presents with classic olecranon bursitis. This is a commonly nonpainful outpouching of fluid due to irritation/bursitis of the olecranon bursa in the posterior elbow, which lies just deep to the skin over the olecranon. Olecranon bursitis is more likely in individuals who lean on their elbows or otherwise experience frequent friction/compressive forces to their posterior elbow. Treatment involves elbow padding as conservative care; if this fails, olecranon bursa aspiration is likely indicated. Corticosteroid injection is not warranted in the bursa, as injection of this superficial bursa with corticosteroid will increase the risk of infection without added clinical benefit compared to aspiration alone. Orthopedic surgery referral is indicated in recalcitrant cases failing aspiration. There is little to no clinical suspicion of an infected bursa in this patient, thus empiric antibiotics alone are not warranted.

A 52 year-old male presents with 3 weeks of anterior right knee pain and swelling. On examination you detect a bulbous, soft, fluid-filled pouch just anterior to the patient's patella, which is tender to palpation. The patient denies fever, chills, rash, or tick exposure. He works in home construction and spends a lot of his time on his hands and knees installing flooring. Strength, sensation, and reflexes are intact in both lower extremities. He asks you to do something today for him, because his pain is unbearable. Which of the following is the most appropriate next step?

This patient presents with classic prepatellar bursitis, the fluid-filled distension of the prepatellar bursa just anterior to the knee cap (patella). This is most often irritated with prolonged anterior knee friction/compressive forces (e.g. prolonged kneeling). Prepatellar bursitis can be painful. Knee pads are a useful conservative treatment for this problem in patients who are willing to trial conservative care alone. This patient requests urgent treatment today; thus, aspiration of the prepatellar bursa is warranted. Corticosteroid injection is not appropriate into this superficial bursa, as this will increase the risk of infection without added clinical benefit. Orthopedic surgery referral for bursectomy is not indicated except in intractable cases.

64 year-old female presents to your clinic with complaints of 2 months of sciatica. She notes burning left buttock pain radiating down her posterior left thigh, stopping at the knee. Her pain is often relieved with shifting her weight onto her right buttock when sitting. She notes some tingling in her left buttock. She denies bowel or bladder dysfunction, and denies saddle anesthesia. On exam she has 5/5 strength in the left lower limb, intact sensation, and normal reflexes. Left slump sit causes left posterior thigh pain. There is tenderness to palpation of the left proximal medial buttock. There is left buttock pain with resisted knee flexion. What is the most likely diagnosis?

This patient presents with classic symptoms of ischial bursitis. The ischial bursa lies between the ischial tuberosity (your "sit" bone on both sides) and the gluteus maximus muscle. It can be inflamed in some patients, causing pain described here, worse with palpation of the ischial bursa (as noted on exam here), and worse with sitting (relieved with shifting weight onto the normal buttock, thus relieving pressure off the inflamed bursa). The ischial tuberosity is also the origin of the hamstrings, and realistically hamstring strains or tendonitis is more likely than ischial bursopathy, but both present very similarly to this patient. Piriformis syndrome would cause lateral buttock pain closer towards the greater trochanter. Greater trochanteric pain would be lateral on the proximal thigh, and SI joint dysfunction would cause a low back pain with positive SI joint tests, none of which were performed in this patient. Treatment of ischial bursitis involves rest, ice, NSAIDs, PT, corticosteroid injection into the bursa.

A 64 year-old female presents to your clinic with complaints of 2 months of sciatica. She notes burning left buttock pain radiating down her posterior left thigh, stopping at the knee. Her pain is often relieved with shifting her weight onto her right buttock when sitting. She notes some tingling in her left buttock. She denies bowel or bladder dysfunction, and denies saddle anesthesia. On exam she has 5/5 strength in the left lower limb, intact sensation, and normal reflexes. Left slump sit causes left posterior thigh pain. There is tenderness to palpation of the left proximal medial buttock. There is left buttock pain with resisted knee flexion. What is the most likely diagnosis? Piriformis syndrome SI joint dysfunction Ischial bursitis

This patient presents with classic symptoms of ischial bursitis. The ischial bursa lies between the ischial tuberosity (your "sit" bone on both sides) and the gluteus maximus muscle. It can be inflamed in some patients, causing pain described here, worse with palpation of the ischial bursa (as noted on exam here), and worse with sitting (relieved with shifting weight onto the normal buttock, thus relieving pressure off the inflamed bursa). The ischial tuberosity is also the origin of the hamstrings, and realistically hamstring strains or tendonitis is more likely than ischial bursopathy, but both present very similarly to this patient. Piriformis syndrome would cause lateral buttock pain closer towards the greater trochanter. Greater trochanteric pain would be lateral on the proximal thigh, and SI joint dysfunction would cause a low back pain with positive SI joint tests, none of which were performed in this patient. Treatment of ischial bursitis involves rest, ice, NSAIDs, PT, corticosteroid injection into the bursa.

A 42 year-old female is evaluated in your inpatient rehabilitation unit. Therapists are concerned that her tremors are interfering with her functional progress. On examination, you notice no tremor at rest, but when you ask her to point to different pictures on a board in front of her, her aim is poor and she often misses her target, overshooting and undershooting it. The therapists have tried wrist weights, and these have not benefited her. What is the next best step for treating her tremor?

This patient presents with tremor during coordination of limbs, notably with attempting to accurately point to a spot in space, and demonstrating dysmetria in the process. This is known as an intention tremor and is associated with cerebellar dysfunction. Failing conservative therapy, oral medications should be initiated to treat intention tremor. Clonazepam has been shown in the literature to be effective for cases of intention tremor.

62 year-old male presents with 1 year of gradual onset low back pain radiating into bilateral posterior thighs, associated with stable bilateral lower extremity paresthesias and weakness. He feels like the longer distances he walks, the heavier his legs become. He denies bowel or bladder changes. On exam he has 3+/5 extensor hallucis longus (EHL) and gastrocnemius strength bilaterally; the remainder of his strength is intact. What is the next best step?

This patient suffers from lumbar spinal stenosis. This is a narrowing of the central spinal canal that leads to compression of the neural structures within; the narrowing is typically due to degenerative spondylosis. The symptoms can be described as neurogenic claudication: leg pain, weakness, leg heaviness with activity, especially those which involve spinal extension, such as standing upright, and walking downhill. Symptoms are typically relieved by spinal flexion, e.g. leaning forward on a shopping cart. Weakness may be demonstrated on exam. The important point to know is that true neurologic weakness by itself (as in this case) does not require emergency spine surgery. This patient has had chronic, unchanged weakness that is not progressing; thus, physical therapy is the most appropriate initial intervention. Acute weakness (e.g. acute, sudden foot drop) requires a stat surgical consult. Lumbar stenosis is best treated with physical therapy and trial of oral medications such as gabapentin. Failing this, MRI of lumbar spine and neurosurgery consult would be indicated, vs. trial of epidural steroid injection (the evidence is mixed for epidural injections in lumbar spinal stenosis).

What does the EAST test ( hands up test) test for?

Thoracic Outlet syndrome

if patient is depressed, name a lab value that you should also screen for?

Thyroid function studies

What is blounts disease? How do you treat it? What will you see ion x ray?

Tibia Vara due to abnormal function of medial portion of teh proximal tibial growth plate and results in bowing in the proximal tibia. common in obese children who walk at 9 - 10 months treat with osteotomy of the proximal tibia and fibula X ray = fragmentation of medial epiphysis of tibia

Tizanidine's mechanism of action

Tizanidine stimulates alpha-2 receptors; this stimulation then inhibits the spinal reflex arc, and is useful for treating spasticity.

A patient presents to your office with neck pain radiating into his right arm and hand in a C6 dermatome distribution. He has no neurologic deficits. He would like to trial physical therapy prior to imaging or more aggressive treatment options, which you agree is appropriate. You write a prescription for physical therapy and would like to trial cervical traction to help alleviate any nerve root compression. He does not have any contraindications to cervical traction. Which of the below parameters are most appropriate for symptomatic relief during cervical traction?

To best relieve cervical nerve root compression, 20-30 degrees of cervical flexion is the optimal angle to open the neuroforamina. For the cervical spine, distraction forces of >25 lbs are optimal, but forces beyond 50 lbs provide no further improvement. For the lumbar spine, >50 lbs of traction force is indicated

What is an advantage of a body powered upper extremity prothesis ?

Training and learnign time to use the device is short. bad: poorer cosmetic appearance, suspension system irriates teh axilla, cable of the device can damage clothes and undergarments.

5 year old boy presents with right knee pain. and a limp. He has no hx of trauma. Physcial examination is remarkable for an abducted and externally roated righ thip. He has significant pain with internal rotation. What is the most likely cause of this boys symptoms.

Transient synovitis most commonly in boys between 3 and 8. Children typically have symptoms for one to 3 days and present with an antalgic gait and an abducted and externally rotated hip at rest. Pain is exacerbated with internal rotation of the hip although pain with motion is less severe than children with septic arthritis. treat with rest nad NSAIDS

6 year old with fracture of the physis, metaphysis. what Salter Harris is this?

Type 2

Which muscle fiber type uses only glycolytic metabolism for energy?

Type 2 B There are 2 primary muscle fiber types in humans. They are categorized according to speed of contraction and sources of fuel. Type 1 muscle fibers are slow-twitch with oxidative metabolic pathways. Type 2 muscle fibers are fast-twitch fibers. The type 2 fibers can then be further divided into fast-twitch with both oxidative and glycolytic metabolism (type 2a) and fast-twitch glycolytic (type 2b).

Which type of osteogenesis imperfecta is lethal?

Type 2 is lethal, and the trait must be carried by both parents

Ankle x ray shows fracture of the distal tibial growth plate. It is a fracture through part of the epiphysis and extending across the physis. what type of salter harris injury is this?

Type 3

Which type of osteogenesis imperfecta is in a child that experiences multiple fractures within the first 1-2 years of life

Type 3

What sensory fiber functions to sense touch, pressure, vibration, proprioception?

Type A beta

What sensory fiber functions to sense sharp pain, light touch, and temp?

Type A delta

What sensory fiber functions to sense dull, achy burning pain and temp?

Type C

What does the united states preventive task force recommend for scoliosis screening?

USPSTF found insufficient evidence to recommend for or against annual screening for idiopathic scoliosis, but many states in the US have legislated mandatory funding of school based scoliosis screening programs.

10 year old girl has a fracture: right supracondylar distal humerus fracture with anterior dislocation of the distal fragment of the humerus. Which nerve could be injured?

Ulnar the flexion type pediatric supracondylar humeral fracture is an uncommon fracture and beleived to injure the ulnar nerve

15 year old teenager reports shoulder pain gradually over time. After her pitches he feels a sensation of coldness in his hand and tingling and burning of the arm and forearm. The pain has been there for many years but has worsened since he started playing basebasll and it is not releived with NSAIDS. The pain is not related to temperature or time of day. Physical exam shows decreased coloring of the hands and difficulty moving the shoulder in any direction. There is no pinpoint tenderness identified. A chest x ray shows no bony abnormalities. Which of teh following is the next diagnsotic test used to evaluate his condition?

Ultrasound symptoms of thoracic outlet syndrome include sharp burning pain of teh upper arm forearm and or fingers and common signs include decreased coloring, temperature and tingling of teh hands. An ultrasound is the most appropriate initial imaging test in patients with suspected thoracic outlet syndrome.

Increased intracranial pressure that displaces the medial edge of the temporal lobe through the tentorial foramen is called:

Uncal herniation

Which fibers transmit poorly localized, dull, visceral pain to the spinal cord?

Unmyelinated C

how often should a prosthetic be exchanged in: a child age 5-12 a child age 12-21 a child age 0- 5

Until age 5, a prosthetic device should be replaced annually. From age 5-12, a prosthetic device can be replaced every 18 months. A prosthetic device can be replaced every 2 years from age 12-21.

19 month old girl with weakness since birth and has been unable to learn to walk. Developing contractures at the knees, hypermobility at the wrists and ankles. What is the diagnosis?

Urlichs congenital muscular dystrophy hypermobility is evident in disorders of collagen such as muscular dystrophies due to collagen 6 mutations. Urlichs presents with neonatal weakness and multiple contractures. Characteristic feature is distal hypermobility with protrusion of the calcanei

During a blink reflex study, you recall that the Vs nucleus is located within which of the following structures?

V s is in medulla The blink reflex study tests CN V and CN VII in the brainstem and peripherally. CN V has two nuclei being tested (Vm and Vs). The Vm nucleus lies within the pons. It accepts CN V input from the face and conducts it to the ipsilateral CN VII nucleus, causing an ipsilateral blink (R1 response). It also simultaneously conducts the impulse to the medulla where the Vs nucleus lies. Vs accepts the impulse from Vm and sends the impulse to the bilateral CN VII nuclei, which then causes a bilateral blink via the orbicularis oculi muscles (R2 response).

When do vertebral compression fractures need surgery?

Vertebral body compression fractures typically occur at the thoracolumbar junction, as in this case. They are typically due to trauma, and osteoporosis is sometimes a contributing factor. Spine surgery is required if the spine is unstable (middle column damage, or any 2 columns being damaged), there is > 50% loss of vertebral body height, or there are neurologic deficits as a result of the vertebral body fracture

VACTERL or VATER syndrome is associated with?

Vertebral defects Anal atresia Cardiac defects Tracheo Esophageal fistula Renal dysplasia limb deficiency

Sheathed tendons such as the flexor tendons of the hand receive blood supply from where?

Vincula

You are performing pre-participation physical examinations on a high school lacrosse player. Vitals signs reveal a blood pressure of 149/81, heart rate of 92, and respiratory rate of 16. Visual acuity is 20/60 on the right 20/80 on the left. Past medical history is significant for two previous concussions without residual symptoms. Medical examination reveals an I/VI systolic ejection murmur that is eliminated with Valsalva, which has been previously evaluated. Musculoskeletal examination is significant for left shoulder pain with empty can testing. Which finding will prevent immediate clearance of this student-athlete to participate in lacrosse?

Vision According to the most recent pre-participation guidelines, an innocent murmur does not indicate heart disease and full participation is permitted. Prior concussions, while concerning, do not warrant restriction if the patient is asymptomatic and has been previously evaluated. A weakened rotator cuff, while worthy of a follow-up examination, does not warrant restriction from competition. Eye sight that is less than 20/40, especially in athletes where depth perception and a moving ball is part of the sport (such as in lacrosse), requires further evaluation prior to clearance

A 68 year-old male is hospitalized for sudden-onset left hemiparesis. On exam you notice his right eye appears to be looking down and to the right. His sensation and coordination are otherwise intact. What is the most likely diagnosis?

Weber (medial midbrain) syndrome is caused by an interruption of the medial midbrain's blood supply (typically from the ipsilateral posterior cerebral artery - PCA), leading to paralysis of the medially located corticospinal tract fibers, which will cause a contralateral hemiparesis, as these fibers have yet to decussate (cross) in the medulla more inferiorly. As we are in the midbrain, medially we have the CN 3 nucleus, thus causing an ipsilateral CN 3 palsy. Medial medullary syndrome typically causes contralateral hemiparesis and ipsilateral tongue deviation towards the side of the lesion.

When treating a patient with a traumatic brain injury (TBI) for detrusor hyperreflexia using anticholinergic agents, what is a possible adverse effect?

Worsening of delirium

What is the most common type of stroke out of the following options? Hemorrhagic Lacunar Thrombotic Embolic

lacunar

what prophtylactic antiemetic can be used safely by a patient with parkinsons controlled on levodopa/carvidopa

Zofran

Zolgensma (onasemnogene abeparvovec-xioi) is an FDA-approved gene therapy to treat which of the following conditions?

Zolgensma (onasemnogene abeparvovec-xioi) was FDA-approved on 5/24/2019 as gene therapy for patients under 2 years age with SMA. This medication is given once intravenously, and is intended to replace the mutated SMN1 gene in these patients, creating functional SMN protein and improving motor function while preventing disability progression.

34 year old man lifted a cenet block and had a radiculopathy with numbness on dorsum of foot and weakness with ankle dorsiflexion. what will you see on EMG after6 months?

large amplitude motor unit action potentials

Kluver-Bucy syndrome

a behavioral disorder caused by bilateral temporal lobe damage. inappropriate sexual behavior. frequently placing objects in mouth

what is osteochondritis dissecans lesion?

a bone or cartilage fragment with diminished blood supply from trauma or vascular insult

A detrusor leak pressure of what value is predictive of upper urinary tract injury and renal decline? places patient at the highest risk for long term renal impairment.

a detrusor leak point pressure above 40 cm of water is predictive of upper urinary tract injury

What is the F wave?

a pure motor late response which occurs after teh compound motor action potential variable response, not a true reflex

What is myelochisis?

a sever form of spina bifida where the involved area is represented by a flattened plate like mass of nervous tissue with no overlyin gmemebrane. the exposure of the nerves makes the baby more prone to life threatening infections such as meningitis

What is the most frequent cause of stroke?

large vessel embolis

ACL tears can be associated with Segund fractures, which is an avulsion fracture of the ?

lateral tibial condyle. X-rays are important to rule out this fracture following an acute ACL tear.

abnormal slowing greater than ??? across the fibular head shows possible impingement

abnormal slowing greater than 6 m/s across the fibular head

Which of the following removes interference associated with stimulus artifact by directly reducing impedence? 1) decreasing stimulus intensity 2) rotating the anode around the cathode 3) removing perspiration, lotion makeup between stimulator and recording electrodes 4) abrading the skin and cleaning the pick up electrodes

abrading the skin and cleaning the pick up electrodes

A 28-year-old woman presents with distal weakness, dysarthria, and dysphagia with facial weakness. Physical examination reveals normal sensation, areflexia, and diffuse weakness. What is the earliest electrodiagnostic finding expected in this patient?

absent late responses

medications helpful in acute phase of CRPS? Chronic phase of CRPS?

acute = IV and oral bisphosphonates chronic = intranasal calcitonin

A 76-year-old woman is referred for rehabilitation following a recent left total hip arthroplasty. To prevent hip dislocation, you instruct the patient to avoid hip

adduction beyond neutral

For a patient with dysphagia following stroke, the chin-tuck maneuver is effective because it decreases

airway opening

The patellar tendon bearing (PTB) socket for a transtibial amputee is designed:

alignment in 5 to 8 degrees of anterior flexion The socket is aligned in approximately 5° - 8° of flexion to increase initial tension on the quadriceps tendon (discouraging knee hyperextension) and enhance weight bearing to the anterior aspect of the residual limb. The PTB socket is designed to accept weight at the patellar tendon, medial flare of the anterior tibia, lateral aspect of residual limb, pretibial muscle mass between the tibial crest and fibula and popliteal fossa

2 meds helpful for fatigue in MS?

amantadine and modafinil (provigil)

28 year old female runner with SI joint pain. Pain has not responded to conservative management. training schedule recently increased due to upcoming marathon. Bone scan reveals radionuclide uptake in the lateral aspect of the right sacrum at the SI joint. What is the greatest risk factor for this patients most likely source of pain.

an increase in impact activity she has a stress fracture

innervation of supinator and anconeus?

anconeus=radial c7 c8 supinator=radial c6 to c8

Fanconi syndrome is associated with?

anemia and leukpenia

which type of sternoclavicular dislocation is most common?

anterior

facial nerve provides taste to what part of the tongue? what dies it do to the salivar glands?

anterior 2/3s parasympathetic but not to the parotid

what is quetiapine?

antipsychotic seroquel

Which neurologic deficit showed improvement with slow repetitive transcranial magnetic stimulation to the un-injured hemisphere in stroke patients?

aphasia Group studies using slow repetitive TMS demonstrated improved language test performance in post-stroke aphasia. Stimulation was applied to the uninjured hemisphere, reducing the risk of treatment-related seizure

possible locations for ulnar nerve entrapment

arcade of Struthers retrocondylar groove acuate ligament of FCU humero ulnar arcade aponeurocis of FCU aka cubital tunnerl

what is Niemann Pick type C

autosomal recessive disorder caused be defects in intracellular cholesterol circulation resulting in lysosomal storage of phospholipids and glycolipid metabolism. aka there is a problem in cholesterol transport and it thereforea ccumulates in perinuclear lysosomes

snapping hip syndrome is more common in which of the following athletes? long distance runners, swimmers, elderly persons, ballet dancers, children?

ballet dancers

The process in which an inpatient rehabilitation unit receives information about outcomes as compared to centers in the geographic region is referred to as:

benchmarking A benchmark is a target value or standard for comparison for a performance indicator. Functional outcomes and efficiency of functional improvement during inpatient rehabilitation admission are examples of performance indicators. The UDS-FIM database provides a means by which individual rehabilitation units can compare their outcomes to other centers across the nation. This process of comparing outcomes to a standard is referred to as benchmarking.

locations for median nerve impingement

bicipital aponeurosis ligament of struthers pronator teres syndrome AIN syndrome wrist

Assuming the use of prostheses, which lower extremity amputation requires the greatest energy expenditure for ambulation? A. Bilateral above knee B. Hemipelvectomy C. Unilateral hip disarticulation D. Below knee plus above knee

bilateral above knee

12 week old has 1 leg longer than the other, ortolani is negative, what is the next step?

bilateral hip ultrasound to look for developmental dysplasia of the hip

what is the site action of pregabalin?

binds aplha 2 delta site of voltage gated calcium channels

How do rocker shoes alleviate pain in osteoarthritis?

by reducing bending forces while the foot is in midstancw

F wave for median neuropathy?

can be prolonged or absent when the condition is severe

prescribe the right cane length with specifics of angle of elbow

cane length should be measured from floor to the greater trochanter or distal wrist crease with elbow in flexion 20to 30 degrees

MOA of carbamazepine

carbamazepine, which acts via sodium channel inhibition on neurons.

13-year-old has recently been diagnosed with limb-girdle muscular dystrophy. Routine evaluation of which organ system is recommended?

cardiac

Boutonniere deformity is injury to what?

central slip extensor tendon injury

most common primary brain tumor in young adults

cerebellar astrocytoma

Immunodeficiency states, both congenital and acquired, increase one's risk for developing which of the following conditions?

cerebral lymphoma

How does clarithromycin affect concomittant fentanyl buccal tablets?

clarithromycin causes suppresion of cytochrome p 450. pairing this with fentanyl buccal tablets may increase fentanyl plasma to potenitally fatally high levels

NYHA class 1, 2, and 3 can perform what METs ​

class 1= over 7 Mets class 2= 5-7 Mets class 3 = 2-5 mets

8 month old girl is diagnosed with developemntal dysplasia of the hip by an orhtopedic surgeon. She has a history of oligohydramnios and fetal breech presentation. Physical examination is remarkable for congenital torticollis and metatarsus adductus. Which of the following is the most appropriate next step in managing this patients hip dysplasia?

closed reduction with hip spica cast closed or open reduction in the operating room under anesthesia with a hip spica cast is the treatment of choice for children with DDH who are older than 6 months at teh itme of diagnosis infants up to 6 months are treated with Pavlik harness. splint which prevents hip extension and limits adduction but permits flexion and abduction.

patient with unexplained iron deficiency anemia who is elderly or post menopausal should have what?

colonoscopy

What is required to obtain a driver's license for an individual with cerebral palsy?

complete a driving eval Driving allows individuals with cerebral palsy increased freedom in their surroundings. The clinician may be asked to assess whether a particular individual is medically fit to drive. Because driving requires multiple factors to work in tandem for success, a complete evaluation is needed. The individual requires adequate strength, speed and coordination of either the upper or lower extremities. Cognitive skills such as concentration, attention, visual perceptual skills, and memory will play a role. The ability to process multiple pieces of information simultaneously and make a quick, safe decision is required. It is helpful to have an adaptive vehicle, but that is not necessary to get a driver's license. It is important for seizures to be well controlled and the requirements vary from state to state (from 3-12 months seizure-free) on when a person with seizures can drive, but being off all antiepileptic medication is not required in most states. There are no federal requirements, only state Motor Vehicle Administration requirements.

In a patient with a suspected cranial injury you perform the elctrodiagnostic blink reflex technique with stimulation over the left eye no waveforms are obtained on the left except a normal right R2. With stimulation over the right eye, all waveforms are normal except for and absent R 2 . where is the lesion?

complete left facial nerve lesion

you perform blink reflex technique, stimulate over left eye and no waveformas are obtained, on the left except a normal right R 2. With stimulation over the right eye, all waveforms are normal except for an absent left R2. where is the lesion?

complete left facial nerve lesion

in a patient with a suspected cranial injury, you perform the electrodiagnostic blink reflex technique. With stimulation over the left eye, no waveforms are obtained bilaterally. With stimulation over the right eye, all waveforms are normal. Where is the lesion?

complete left trigeminal nerve lesion

Holt-Oram syndrome is associated with?

congenital heart disease like atrial septal defects and tetralogy of fallot

18 year old has a volar plate avulsion fracture that involves 20% of teh joint surface. How should this injury be treated?

conservative mangement with splinting and hand exercises in order to prevent stiffness most volar plate fractures that involve less than 30 percent of the joint surface may be treated with conservative mangement without surgery. S spiral fracture would be an example of a finger fracture whioch should be sent to a hand surgeon,

What is Emery Dreifus Muscular dystrophy?

contractures of the elbows, ankles and neck. muscle weakness in th eupper arms and shoulder girdle muscles first adn alter teh pelvic girdle and distal leg muscles. CArdiac conduction defects, requiring pacemaker placement. There is susually family history of early deeath due to cardiac disease in males. protein deficiency of EMD x linked form and autosomal dominant form. Treat with excercises

In a patient who sustained a thalamic stroke, how are pain and temperature perception affected below the level of the lesion?

contralateral loss of temperature

Baller Gerold SYndrome is associatged with?

craniosynostosis

what is a bad side effect of risperidone

neuroleptic malignant syndrome

list some causes of extrinsic / chest wall disorder

neuromuscular disorders obesity

What type of electromyographical dsicharge patters is though to originate pre synaptically as opposed to post synaptically at the muscle memebrane?

neuromyotonia

PAtient with saturday night palsy for 2 weeks presents for EMG. What will you see on motor NCS?

decrease in amplitude when radial nerve is stimulated above the spiral groove.

Marfan syndrome is caused by?

defect in FBN1 gene for fibrillin

signs of CN 3 involvement in stroke

deviation of ipsilateral eye to lateral side because of unopposed action of lateral rectus muscle. Ptosis, mydriasis and paralysis of accomodation due to interruption of parasympathetic fibers in CN 3

What is Fukuyama type congenital muscular dystrophy and how does it present?

disorder: autosomal recessive, due to chromosome 9 mutations affecting gene for fukutin. The enzyme is assoicaited wiht the Golgi complex and glycolysates other proteins. presents: diffuse weakness, early contractures of the hip, knees and ankle, intellectual disability, seizures, skull asymetry and cerebral malformations infants have normal strength with progression of teh disease and death in early childhood

The majority of spinal cord injuries due to recreational sports as reported by NSCIS database are accounted by???

diving

7. Pharmacologic strategies employed to improve arousal for patients in a minimally conscious/vegetative state after severe traumatic brain injury (TBI) would most likely include stimulation of

dopaminergic pathways

What is Claude syndrome?

dorsal red nucleus and third nerve fascicle lesion in the left midbrain ipsilateral third nerve palsy and contralateral cerebellar signs

What opioid produces the greates incidnce of delayed respiratory depression?

epidural preservative free morphine due to rostral spread in the cerebrospinal fluid and slow penetration into the brainstem

The fibularis peroneus, brevis and tertirus all function to ?

evert the foot

an active compression test is utilized in order to evaluate suspected patients for labral tears. Which of the following accurately describes a positive active compression test result therefore indicating a labral tear? expression of pain in a patient during internal rotation of the shoulder and pronation of the forearm? Deep pain in the shoulder during contraction of teh biceps when the shoulder is maximally internally rotated and the forearm is in pronated position.

expression of pain in a patient during internal rotation of the shoulder and pronation of the forearm the other option is biceps load test

In a patient with ankylosing spondylitis, which specialist should you refer the patient to for further evaluation and treatment?

eyes Ankylosing spondylitis may have extraarticular comorbidities affecting the eye, gastrointestinal tract, skin, heart, lung, and kidneys. Ophthalmological comorbidities, including conjunctivitis, uveitis, iritis, and iridocyclitis, are the most common, with iritis occurring in up to 40% of patients with ankylosing spondylitis. All patients should be referred to an ophthalmologist for further evaluation and treatment.

What is the most common cause of emergency room visits for traumatic brain head injuries?

falls In 2014, falls became the leading cause for TBI. Falls account for 48% of all TBI related emergency department room visits. Falls disproportionately affect children and older adults. Being struck by or against an object is the second leading cause of TBI-related emergency department visits. Falls and motor vehicle crashes were the leading causes of all TBI-related hospitalizations.

An 80-year-old female with osteoporosis and dementia trips and falls onto her hip. The patient was found on the ground with her limb in an externally rotated, abducted, and shortened position. X-rays of the hip most likely shows:

femoral neck

how do you diagnose neiman pick type C

filipin test shows impared ability of cultured fibroblasts to esterify cholesterol

The majority of burns result from? fire scald electrical chemical

fire

baxters plexus in an entrapment of what nerve and at what level?

first branch of lateral plantar nerve between the abductor hallucis and quadratus plantis muscle

first line treatment for CTS is what? unless you find what on physical exam?

first line is splint unless you find weakness. If you find weakness then they must have mod to severe CTS and they should get an EMG

function of sartorius

flexes, abducts, and laterally rotates thigh; flexes knee

Which of the following is the most common non-traumatic cause of ulnar neuropathy at the wrist?

ganglion cyst

WHat is Alexander disease?

gene mutation for glial fibrillary acidic protein (GFAP). Progressive disorder of astrocytes. 3 forms. Infantile form, which includes megalenceohaly, developemetnal delay, seizures, psyhcomotoe retardation, spasticity and quadraparesis. The juvenile form emerges in childhood and has more significant bulbar symptioms. the adult form displays signs, hyperreflexia, dysautonomia ataxia and sleep apnea. MRI shows diffuse white matter signal hyperintensity mainly in the frontal lobes and anterior cerebral regions.

most common malignant brain tumor in adults

glioma ( high grade astrocytoma) glioblastoma is the highest grade aka poor prognosis

What are trophic changes in CRPS?

hair, nail, skin changes

A 22 year-old intoxicated male falls off a roof at a party and suffers an odontoid (dens) fracture in which the tip of the dens is fractured. What is the most appropriate management for this fracture?

hard cervical collar

most common symptom of metastatic brain lesion

headache

Medulloblastoma is associated with

high incidence of neurocognitive deficits. Medulloblastoma is the most common primary malignant tumor in children. Over 85% of children with this type of tumor have long-term learning and memory deficits. The highest incidence of medulloblastoma occurs in children less than 10 years old. These tumors arise in the cerebellum and have a long-term survival rate of approximately 80%.

What is the highest level of education for the majority of patients with spinal cord injuries?

high school degree

Function of glut max?

hip extension and lateral rotation ( external rotation)

what med can cause drug induced lupus? What antibodies are positive in Drug induced lupus erythematous?

hydralazine and procainamide antinuclear antibodies can be positive in DILE and SLE but antihistone antibodies are specific for DIL

Convert hydromorphine to Morphine

hydromorphone is about 3 times stronger than morpine

Parkingosns usually has what type of dysarthria?

hypokinetic

What is the initial treatment plan for pain in a patient with acute spinal pain from bony metastasis?

ibuprofen

24 year old female runner, has right groin and upper leg/thigh pain, worse with running. MRI of L spine ok, hip x ray ok, no neurological deficits, FADIR is painful, pain wiht passive hip extension, when patient sits in chair and lifts her right led while knee extended, this reproduces pain. What is diagnosis?

iliopsoas tendonitis

WHICH OF THE FOLLOWING DESCRIBVES WERNICKE APHASIA? - impaired speechcomprehension -impaired speech production -impaired speech comprehension an dproduction -slower speech patterns

impaired speech comprehension

what is the primary physiological adaptation taking place with aerobic exercise?

improved AVO2difference aerobic conditioning improves vital signs and overall health by increasing AVO2 difference. aka improved muscular conditioning means the muscles absorb an increased amount of oxygen from the arterial circulation.

blink reflex: right delayed R 1, delayed ipsilateral R2 and contralateral R 2 left: normal R 1, normal ipsilateral R 2 and contralateral R 2

incomplete right trigeminal lesion

Increasing the camber of a wheelchair will result in which of the following:

increased propulsion efficiency

MOA of donepezil

increases acetylcholine in the brain by inhibiting its metabolism, leading to elevated acetylcholine levels in the cortex. cholinesterase inhibitor.

22 week old infant has 1 week onset diffuse hypotonia, ptosis, dysphagia, weak cry, dilated sluggish pupils. infant has been constipated, poor suck. EMG with 20-50 Hz stimulation, reversal of presynaptic block then gradual increase in size of motor unit potentials. What is diagnosis?

infantile botulinum

13 year old gymnast has 6 weeks of dorsal wrist pain, worse with activities like vaulting, what is the mechanism of injury?

injury to the distal radial phsysis pain with wrsit extension and dorsal pain made worse with stress loading.

Balley dancer has popping sensation over teh anterior aspect of the right hip with pain? What is this and how do you treat?

internal snapping hip movement of teh iliopsoas tendon over the femoral head. Treat: activity modification. if interfering with ADLs, PT and CSI

4 year old boy has 1 week of low back pain, normal vitals, on physical exam he is uncomfortable and refuses to bend forward. Loss of lumbar lordosis. What is the cause?

intervertebral discitis typically in children younger than 5 years. gradual onset of pain. no fever. diagnose with MRI.

43 year old women with MS reports bothersome urgency, frequency, and urge incontinence despite 10 mg of oxybutinin three times a day. The patient has also tried trospium with minimal benefit. Urodynamic testing reveals bladder overactivity with normal compliance, minmal post void residual and synergic external sphincter. WHat is the next astep in management?

intradetrusor injection of 200 units of botox. THe patietn has failed 2 anticholinergic medications and would most likely benefit from intradetrusor injections of 200 units of botox.

Which recreational activity is contraindicated following a total knee arthroplasty? A. Hiking B. Power walking C. Jogging D. Cycling

jogging

In a 7 year old why is a knee disarticulation better than a transfemoral amputation?

knee disartic is favorable because of teh residual limb to allow for more weight bearing on the femoral condyls and thus more stability. Bony overgrowth is prserved in knee disartic due to preservedc cartilage. Cosmetic options for this knee are unfavorable though

65 year old female patient is evaluated after the acute onset of diploplia with drooping eyelids. Exam is significant for ptosis and a large unreactive pupil on the left. Her left eye rests in an abducted position with intorsion. She has an intention tremor on the right and an ataxic gait. Where is the patients lesion?

left midbrain claude syndrome

the goal for intracranial pressure after TBI is?

less than 20 mm Hg

with no traumatic event, in an adult, an avulsion fracture of what part of the lower extremity is considered pathognomonic for a pathological fracture?

lesser trochanter primary or metastatic tumor 2 primary causes of avulsion of the lesser trochanter: strenuous flexion of the hip in adolescent athletes with open epiphyses and pathological fracture associated with metastatic lesion of the bone.

Which agents used for post-traumatic seizure prophylaxis is the best choice for a patient with thrombocytopenia?

levetiracetam Bone marrow suppression, decreased platelet production, and thrombocytopenia are common side effects of valproic acid. Carbamazepine and phenytoin are known to cause antibody-mediated thrombocytopenia through increased platelet destruction. Levetiracetam rarely causes thrombocytopenia, though a few isolated case reports exist.

What is the most important factor to consider when injecting myofascial trigger points? A. Type of medication injected B. Localization of trigger points C. Frequency of injection D. Size of needle

location of trigger points

Brain metastases in both men and women are most commonly caused by which primary cancer?

lung

criteria for diagnosis of malnutrition per 2012 consensus statement?

malnutrition should be diagnosed when 2 of the following are present insufficient energy intake, weight loss, ,oss of muscle mass, loss of subcutaneous fat, diminished functional status as measured by hand grip strength, or localized or generalized accumulation that may mask weight loss

16 year old girl is tall fo rher age, has pectus excavatu, generalized ligmantous laxity, arm span 5 incges longer than height, scoliosis. what does she have?

marfan

MRRIage rate and divorce rate for ACI patients?

marriage rate is below and divorce rate is higher thna the general population

what does "sign of benediction" look like?

median nerve inury results in "sign of benediction. This is due to the loss of innervation of teh lateral 2 lumbricals of the hand and the lateral half of teh flexor digitorum profunmdus which are supplied by the median nerve. Flexion at the proximal interphalangeal joints of digits 4-5 is weakened, but flexion at the metocarpophalangeal joints and distal interphalangeal joints remains intact. The extensor digitorum is left unnopposed and teh metacarpophalangeal joints of digits 2-3 remain extended while attempting to make a fist.

most common brain tumor in less than 7

medulloblastoma

25 year old runner with no medical history reports pain in the ball of her foot for 3 months that also radiates to her toes occasionally. It is mitigated by rest and aggravated by running. The pain occurs at all times of teh day and is not related to psychological stress, sleep or temperature. She recently started running a lot more daily and noticed that her new shoes seemed tight. She has tenderness over the ball of the foot and the metatarsals

metatarsalgia

25 year old runner with no medical history complains of pain in the ball opf her foot. She has been experiencing this pain for 3 months, the pain also spreads to her toes occasionally. It is mitigated by rest and relaxation and is aggravated by running. The pain occurs at all times of teh day. She recently started running for longer periods every day and noticed that her new shoes seem a little tight. vitals stable. Physical exam shows tenderness over teh ball of the foot and over the distal metatarsals. her foot also appears to be quite arched. The patient has no trouble putting weight on the foot and does not have any discoloration. What is the best step in management. ?

metatarsalgia pads pain on the metatarsal heads that result in tenderness over teh ball of the foot adn sometimes the toes. Patients who run frequently or who wear tight fitting shoes often complain of theses symptoms. condition presents with arching and excess pronation of the foot. usually self limited.

Which pharmacologic agent should be avoided because of its cognitive side-effects in individuals with brain injury? metoclopramide ( reglan) omeprazole ( prilosec) Erythromycin Sucralfate

metoclopramide ( reglan) The use of metoclopramide (Reglan) should be avoided because it is known to cause sedation and significant cognitive difficulties for individuals with brain injuries, especially for those regaining consciousness. It also has potential side effects of extrapyramidal movements and tardive dyskinesia. The other agents do not have significant cognitive effects on brain-injured individuals.

csf findings of someone with MS

minimally elevated to normal cell count normal total protein elevated IgG presence of oligoclonal bands

Name an Absolute contraindication to exercise stress test

moderate aortic stenosis active endocarditis aortic dissection hemodynamically unstable

H reflex is what kind of reflex?

monosynaptic

WHile ischicontainment sockets are usually favored, what is an advantage of quadrilateral femoral sockets?

mores successful in patients with intact hip adductor muscles

describe martin gruber? what will you see on NCS?

motor median to ulnar innervation you will see a larger response on the proximal median motor study because there will be more motor fibers aka more axons available in proximal stimulation than in distal stimulation. fast conduction velocity positive initial deflection on motor study

developmental dysplasia of the hip is associated with what muscular abnormality?

muscular torticollis

myasthenia gravis (MG), which is due to antibodies against ?? Lambert-Eaton Myasthenic Syndrome (LEMS) is due to antibodies against ???

myasthenia gravis (MG), which is due to antibodies against postsynaptic neuromuscular junction (NMJ) acetylcholine receptors. Classically symptoms (proximal muscle weakness) worsen with exercise, as the day progresses, and are associated with diplopia. Lambert-Eaton Myasthenic Syndrome (LEMS) is due to antibodies against presynaptic calcium channels, and p

Which of the following is the most common presenting symptom in patients with acquired cervical stenosis?

neck pain

Bradycardia, hypotension, temperature dysregulation with resultant hyperthermia are signs of?

neurogenic shock

Which of the following would typically be found in the transfemoral prosthesis of a 7-month-old amputee?

no knee component Pediatric amputees are prescribed a prosthesis when they are at least 6 months old, and are starting to pull themselves to stand (fit to stand), in keeping with the timeline for developmental milestones. For a child learning their initial balance and ambulation, a simple and stable prosthesis is best. Progressing proximally to distally, the transfemoral prosthesis would have a solid ankle foot and no knee joint, since children cannot control the movement in an articulated knee until the age of three years, and an articulated foot will make balance difficult to learn and increase fall risk. The transfemoral socket is placed in abduction to match the increased hip abduction and external rotation found in children of this age. A total elastic suspension (TES) belt is the simplest for parents to learn, and is more easily adjustable as the child grows, and allows for (relatively) greater ease during diaper changes.

Name a risk factor that is associated with development of a venous thromboembolism (VTE) in a child with traumatic brain injury (TBI).

non accidental trauma older age of 15-20 years (adjusted odds ratio [aOR] 3.7, 95% CI 1.8-8.0), venous catheterization (aOR 3.0, 95% CI 2.0-4.6), mechanical ventilation (aOR 1.9, 95% CI 1.2-2.9), tracheostomy (aOR 2.3, 95% CI 1.3-4.0), nonaccidental trauma (aOR 2.8, 95% CI 1.1-6.9), increased length of stay (aOR 1.02, 95% CI 1.01-1.03), orthopedic surgery (aOR 2.4, 95% CI 1.8-3.4), and cranial surgery (aOR 1.8, 95% CI 1.1-2.8).

melodic intonation therapy can be useful for patients with what type of aphasia?

non fluent aphasia like Broca aphasia melodic intonation therapy recruits the right hemisphere by incorporating melodies or rhythms into communication

4-year old boy presents with acute left elbow pain. The pain began approximately 30 minutes ago when he was playing at the playground with his father. His father is present and states, "I'm not sure what happened. I picked him up by his hands and started swinging him around, which he usually loves, but today he started screaming and crying right away, as soon as I picked him up." On exam, the boy is neurologically intact, but refuses to let you examine his left elbow, keeping it close to his side. You observe no deformity in the left elbow. Which of the following would you expect to find on a left elbow x-ray of this patient?

normal This patient presents with classic Nursemaid Elbow. Nursemaid Elbow is a radial head subluxation and relocation due to sudden distally directed axial force upon the radius, which entraps the radial head annular ligament within the radiocapitellar joint of the elbow, causing severe elbow pain. X-rays of the involved elbow are typically normal and are generally not required in patients under 5 years of age, as Nursemaid Elbow (pulled elbow) is a clinical diagnosis. Treatment involves prompt closed reduction with hyperpronation of the involved forearm or supination and elbow flexion of the involved upper limb.

normal 25- OH vitmmain D levels in children? insufficiency? deficiency?

normal = > 20 insufficiency = 12-20 deficiency = < 12

ABI, what is normal? TcPO2 ( transcutaneous pressure of oxygen) : what is normal?

normal ABI is greater than 0.9 TcPO2, greater than 50

18 year old, left knee pain, fevers, chills, non pruritic rash, no urinary symptoms, vitals normal, violaceous papules on palms, soles and lower legs. left knee and right achilles tendon are tender to touch. CBC CMP normal. What is the next step in managmenent?

nucelic acid amplification testing for neisseria Gonorrhea

Phantom limb pain is associated with ? age ? gender ? type of amputation

older age, female, transfemoral, lack of social support prior to amputation

When different types of prosthetic feet are utilized, the main adaptations were prevelant where?

on the intact limb rather than the limb with the prosthesis

Convert Norco to Morphine

one to one

Name 5 domains of MMSE

orientation recall attention calculation language manipulation and constructional praxis

A 27-year-old golfer presents with pain on the volar ulnar aspect of his hand. X-ray shows he has a fracture of the hook of his hamate bone. What is the treatment of choice?

ortho referral Hamate hook fractures represent 2-4% of all carpal fractures and are a common injury in golf, baseball and hockey. Patients usually present with vague complaints of pain at the volar ulnar aspect of their hand with pain provoked when attempting a tight grip. Plain radiographs may not visualize the fracture thus a CT scan should be considered if one has a high suspicion for fracture. Excision of the hook of the hamate is considered the treatment of choice. Acute injuries and non-displaced fractures may be treated non-operatively but excision is the treatment of choice. Nonunion rates greater than 50% and as high as 80-90% can occur with conservative treatment. Therefore, all hamate hook fractures should be referred to a hand surgeon for possible surgical intervention.

34 year old female sawimmer has dull aching pain in her groin tha is worse with swimming. Tender to palpation of adductor longus origin. X ray shows widening of the physis. what is the diagnosis and what other diagnosis whows widening of the physis?

osteitis pubis and pregnancy

58 year old male presents with right knee efusion. He denies any trauma or injury but has had some knee pain in th epast. he woke up with a swollen knee and pain this morning. He has no fever or chills. 3+ effusion, mininmal range of motion\ arthrocentesis shows yellow color, high viscocity, WBC per mm 1,500 and PMNs 20%. What is the most likely diagnosis?

osteoarthritis WBC less than 2000 mm3 and teh absence of crystals and the absence of fever are charactersitic of osteoarthritis knee effusions.

what benign bone tumor is most common in long bones such as the femur and tibia, most commonly seen in the posterior elements of the spine, pain classically worse at night and relieved with NSAIDS, x rays show a sclerotic lesion with an occasional central lucency or nidus. NO cortical disruption or periosteal reaction

osteoid osteoma osteosarcoma would be aggresive lytic lesion on x ray with florid periosteal raction, sunburts , codman triangle,.

What is the most common type of voiding dysfunction among elderly individuals?

overactivebladder Overactive bladder, also referred to as detrusor overactivity, is the most common type of voiding dysfunction among incontinent elderly individuals. Outlet obstruction (caused by prostatic hypertrophy) is the 2nd most common cause of incontinence among elderly male individuals, while stress incontinence is the second most common cause of incontinence among elderly females. Detrusor underactivity is much less common, but may contribute to overflow incontinence in this age group.

Convert oxy to morphine

oxy is 1.5 times stronger than morphine

When combined with active range of motion (ROM) exercise, which modality can improve hand function in individuals with scleroderma (systemic sclerosis) or rheumatoid arthritis (RA)? A. Transcutaneous electrical nerve stimulation B. Therapeutic ultrasound C. Cryotherapy D. Paraffin baths

paraffin baths

What is the final activating force used in the muscle energy technique? A. Patient muscle contraction B. Movement to the barrier C. Position hold by the practitioner D. Low-amplitude, high-velocity thrust

patient muscle contraction

a 21 year old rugby player had a sudden onset of chest pain after tackling an opposing player. HE denies any dyspnea. He has no history of health problems. On physical exam, he has no pain or derformity in his clavicle. His roator cuff muscles are intact. There is an easily seen and palpable defect in th emuscle belly of his chest, inferior to his clavicle. What is the most likely diagnosis?

pectoral muscle tear

brachial and lumbosacral plexopathies are a form of?

peripheral neuropathies

Which phase of cardiac rehabilitation is a class 1 recommendation from the American Heart Association and American College of Cardiology after myocardial infarction?

phase 2 Referral to outpatient phase II cardiac rehabilitation is a class 1 recommendation from the American Heart Association and American College of Cardiology after myocardial infarction, coronary vascularization, valvular heart surgery, heart transplant, and in stable heart failure with reduced ejection fraction

What is the leading cause of death for individuals with subacute or chronic traumatic spinal cord injury?

pneumonia Data from the National Spinal Cord Injury Statistical Center Database demonstrate that the leading cause of death following traumatic spinal cord injury (SCI) is non-circulatory respiratory disease, with the large majority representing deaths due to pneumonia. Heart disease is the second leading cause of death, followed by septicemia and other infective diseases. Pulmonary embolism is one of the leading causes of death in the first month after injury, but only the 5th leading cause of death in subacute or chronic SCI.

list 3 types of inflammatory myopathies

polymyositis dermatomyositis inclusion body myositis

dysarthria and clumsy hand syndrome is due to a lesion at the ?

pons, anterior limb of internal capsule, corona radiate, putaminal hemmorrhages

Pitchers with glenohumeral internal rotation deficit is secondary to what?

posterior capsular tightness

Radiographic visualization of what on x ray woukld indicate an occult elbow fracture?

posterior fat pad

which artery provides 2/3 of the blood supply to the humeral head!

posterior humeral circumflex artery

Manual therapy utilizes instructions and maneuvers to achieve maximal painless movement of the musculoskeletal system with a goal of improving:

postural balance

Following a burn injury, transparent custom total contact face mask orthoses are used to A. cover the disfigurement B. preserve facial contours C. increase eyelid eversion D. prevent infection

preserve facial contours

medial circumflex femoral artery arises from what blood vessel?

profunda femoral artery

According to the American Stroke association guidelines, what is true regarding the guidelines for the use of anticonvulsants in patients with intracerebral hemorrhage.

prophylactic anticonvulsant use is not recommended

IN a child with DMD, what muscle group is most weak at time of presentation

proximal lower extremities

When providing patient education to a 23-year-old woman with a complete T4 spinal cord injury (SCI), you should tell her that she will be unable to experience

psychogenic vaginal lubrication

A person who had a lacunar infarct in the posterior limb of the internal capsule would most likely present with

pure motor stroke

What are the mechanical differences present in a wheelchair which is prescribed with the intention of being raced in athletic competitions?

racign wheelchairs have have larger wheels usually around 27 inches set low to the top -small hand rim ( 12 inches) -narrow pneumatic tires (160 PSI) -12-18 inches diameter casters with steering handles and brakes installed on the front wheel. -The real wheel in a standard wheelchair measure 24 inches and a quick release mechanism may be present in order to load the wheelchair onto a vehicle with ease.

Which of the following is the most frequent association of nerve injury and fracture?

radial nerve and humeral fracture Radial nerve injury with humeral fracture is the most frequent association of nerve injury and fracture. Radial neuropathy occurs in 11% of humeral fractures, but up to 60% of midshaft frac-tures. Especially in multi-trauma scenarios, peripheral nerve injury may not be identified on initial evaluation.

13-year-old right hand dominant baseball pitcher is seen in your clinic with right lateral shoulder pain that is aggravated by throwing. He denies any prior shoulder injury. Your examination is significant only for tenderness along the lateral deltoid. Radiographs of right shoulder are normal. The next best step in management of this patient is:

radiograph the left shoulder This adolescent has the hallmark history and potential signs of Little League Shoulder (LLS)/Glenohumeral Epiphysiolysis. LLS is a repetitive stress injury due to overuse and improper rest that typically occurs in young teenagers. It is likely a result of torsional overload of the proximal humeral epiphysis during maximal shoulder external rotation during the throwing cycle. In order to fully compare the physes, contra-lateral radiographs are needed as even subtle differences can make the diagnosis. MRI imaging not as efficient and more expensive. Treatment recommendations include elimination of throwing for 6 weeks after diagnosis and an additional 6 weeks of rehabilitation without throwing. A return to throw/pitch program is initiated once the patient is pain free and has completed a rehabilitation program without setbacks.

What is the primary disadvantage of moving the rear axle of a wheelchair forward?

ramps can be difficult Moving a wheelchair's rear axle forward enables the user to propel the chair with less muscle effort and fewer strokes. Because the modification causes more weight to be centered over the rear wheels, it is easier to pop a wheelie, negotiate obstacles and ascend or descend curbs. However, moving the axle forward can also make the wheelchair more likely to tip backwards thus making it more difficult to propel the chair up a ramp.

THe Duncan test or Ely test is used to asses what?

rectus femoris muscle

Nerve conduction findings in patients with critical illness polyneuropathy include

reduced sensory and motor evoked amplitudes

which is more liekly to result in a neurologically incomplete injury? bilateral cervical facet dislocation,retropulsed disc, gunshot injury with transcanal bullet location. or thoracic rotation injury

retropulsed disk

what is teh optimal post operative dressing for a transtibial amputee?

rigid removable dressing

What is the most commonly used switch access method for alternative and augmentative communication devices? A. Picture text scanning B. Noun verb scanning C. Row column scanning D. Line checkbox scanning

row column scanning

What are the 4 categoris in the budapest criteria for CRPS?

sensory vasomotor sudomotor motor/trophic

Which of the following may develop from exposure to cisplatin?

sensory impairment Cisplatin is a neurotoxic chemotherapeutic agent used in a variety of cancer types. Its putative mechanism of action with respect to neuropathy is disruption of cellular functions at the dorsal root ganglion and subsequent death or dysfunction of the sensory nerves. Since platinum analogues do not usually cross the blood-brain barrier at contemporary doses, the anterior horn motor cells are unaffected. Abnormal sensations (paresthesias), painful sensations (dysesthesias), loss of sensation (anesthesia), and other neuropathic sensory disorders are common, but weakness is not generally seen. Gait dysfunction and ataxia is from sensory dysfunction and not muscle weakness.

With TBI, severe disability unlikely when coma lasts less than ? weeks With TBI, Good recovery is unlikely if coma last greater than ? weeks Severe disability is unlikely if duration of PTA is less than

severe disability unlikely when coma lasts less than 2 weeks Good recovery is unlikely if coma last greater than 4 weeks ? Severe disability is unlikely if duration of PTA is less than 2 months

In evaluating a patient with pain, you decide to squeeze her left tibia together by wrapping your hands around the medial and lateral borders of the tibia and squeezing. The purpose of this test is to diagnose which of the following conditions?

shin splints Squeezing the tibia and reproducing a patient's pain is indicative of shin splints (medial tibial stress syndrome). One may also perform a squeeze test to examine a high ankle sprain, but x-rays would be the best test for Maisonneuve fracture (a complication of a high ankle sprain), not physical exam.

What is another name for CRPS?

shoulder hand syndrome

initial treatment for non displaced scapula fracture?

shoulder sling for 2 weeks then early ROM

Which assessment tool can be used to evaluate functional limitations in patients with myofascial pain?

sickness impact profile

what stage of crps pain decreases, trophic changes occur, hand skin appear pale and cyanotic with a smooth shiny appearance, feeling cool and dry, bone demineralization progresses with muscular weakness/atrophy, contractures/flexion deformities of shoulder/hand , tapering digits, no vasomotor changes

stage 3

patient injured his elbow and kept it in a sleeve for 8 weeks an d now has limited ROM. He goes through 8 weeks of therapy and still has limited ROM. What is next?

static progressive elbow casting

A 21-year-old basketball player attempts to catch a pass from his teammate but drops the ball and immediately experiences pain in his distal third digit. Instant replay shows the basketball hitting his third fingertip causing a forced flexion moment at the DIP. He is taken out of the game and seen by the team physician; during initial examination, the patient is unable to actively extend the DIP of his third finger. Xrays are negative for acute fracture. What splint is most appropriate to promote proper healing of his injury?

stax splint The vignette describes a mallet finger injury, which is often seen in sports such as basketball or baseball. Typically, a ball hits the distal aspect of a finger, causing flexion of the DIP and resultant extensor tendon rupture. This leads to inability to actively extend the DIP. X-rays are appropriate to rule out avulsion fracture. A stax splint or DIP extension splint is appropriate to allow for healing of the DIP extensor tendon. A swan-neck ring splint and Boutonniere ring splint are examples of static-motion blocking splints that are used to treat their respective namesake. A resting hand splint promotes ROM of joints of the hand to avoid contracture formation, typically after stroke.

what med can be given following whole brain irradiation in order to reduce the expected brain swelling that occurs as a result of radiation

steroids

Hemiplegic cerebral palsy is often due to?

stroke which may result from placental embolisms

What does a bipartate patella look like on x ray?

superior lateral ossification center fails to fuse, . There are irregular margins and rounded edges. Charactersitic superior lateral location

injury to the suprascapular nerve at the spin glenoid notch will result in weakness in what shoulder functions

supraspinatus and infraspinatus will be injured so weakness with shoulder abduction and external rotation

You are reviewing left lower limb EMG/NCS findings and trying to propose a possible treatment plan to the patient. The patient is experiencing foot pain not relieved by NSAIDs or heat/ice. NCS reveals normal superficial fibular and sural SNAPs (sensory nerve action potentials), prolonged latency of medial and lateral plantar nerve SNAPs, and abnormal CMAP (compound muscle action potential) to abductor hallucis (AH). The CMAP to the extensor digitorum brevis (EDB) is normal. Needle EMG reveals decreased recruitment in AH, abductor digiti quinti pedis (ADQP), and lumbricals. EMG of tibialis anterior, EDB, gastrocnemius, fibularis longus, hamstrings, rectus femoris, tensor fascia lata, and lumbosacral paraspinals is normal. Which of the following is the most reasonable treatment strategy for this condition?

surgery These findings suggest tarsal tunnel syndrome, a rare compression of the tibial nerve as it passes through the tarsal tunnel around the medial malleolus, underneath the flexor retinaculum. The flexor retinaculum is a common culprit that compresses the tibial nerve too tightly, causing the patient's symptoms of plantar foot pain with numbness/tingling and possible foot muscle weakness. Surgical release of the flexor retinaculum (akin to carpal tunnel release) is the best option of these choices (a structural solution for a structural problem). US-guided tarsal tunnel injection could also be considered. Bracing would not do anything to decompress the tarsal tunnel. Gabapentin also would not decompress the tarsal tunnel, but mask the symptoms while the nerve compression continues, potentially destroying axons over time. Peripheral nerve stimulation is not appropriate for this reason also.p on

what is sarcopenia?

syndrome defined by the presence of the loss of muscle mass, strength and performance. associated with increased rates of functional impairment disability and falls. Its presence carries a mortality risk

the potential for lower extremity recovery improvement correlates well with?

the level of injury in complete paraplegics some recovery is seen in 15% for &T9-T11 versus 55% for T 12 and below

When the lunate doslocates anteriorly, it can compress what nerve?

the recurrent branc of teh median nerve leading to altered sensation over the palmar aspects of the opponens pollicus, abductor pollicus brevis and the flexor pollicus brevis

While performing an electromyography examination on fifty seven year old male with complaints of back pain, you note a waveform that starts with an initial positive deflection followed by a quick uprising while testing a muscle. You notice the waveform in another quadrant during studying of teh same muscle and again in two separate muscles during exam and that it fires regularly. What does this potentially signify.

this describes a fibrialtion potential which will alwys have aninitial positive deflection, regular rhytm, and sound like rain on a tin roof. They are generated by a denervated single muscle fiber.

most metastatic lesions occur in what part of the spine

thoracic

In which of the following cases is microwave diathermy most appropriate?

to hasten the breakdown of a hematoma Ultrasound uses ~1 MHz sound waves to produce vibrations below the skin and produce images out of those vibrations and sound wave reflections; it is most useful to heat deep tendons, ligaments, and for general MSK pain (up to 8cm below the skin). Short wave diathermy uses radio waves ~27 MHz to heat up tissues up to 5cm deep. Microwave diathermy uses microwaves ~1000 MHz to heat very superficial structures 1-3 cm deep, and is indicated to hasten the breakdown of hematomas. Diathermy is contraindicated over metal and cancer especially, while ultrasound is not appropriate near electrical implants or cancer.

how do you treat Osgood schlater?

topical ice therapy, protective pad, short course of ibuprofen and the physical therapy

Which cervical injection technique has been associated with acute catastrophic neurologic injury?

transforaminal epidural steroid injection

5 year old with right sided limp, hx of recent URI with fever, antalgic gait. what is this diagnosis and what is the next step in management?

transient synovitis NSAIDS

The type of muscle fiber that purely utilizes glycolysis, not aerobic respiration, for energy is:

type 2 b Type I muscle fibers are slow twitch, "red" fibers that utilize oxidative phosphorylation as part of aerobic respiration. These are the muscles primarily utilized in cardiovascular endurance exercises such as running a marathon. Type IIa fibers are fast twitch, "white" muscle fibers that are anaerobic and utilize glycolysis primarily for energy. Type IIa fibers also do have some degree of aerobic capacity. Type IIb fibers are purely glycolytic and have no aerobic capacity; these are used chiefly in powerful fast twitch activities, such as sprinting or powerlifting.

Which modality most effectively treats the periarticular tissue of the hip joint?

ultrasound

Which modality should precede shoulder stretching in a patient with adhesive capsulitis?

ultrasound

You are called to the bedside on PM&R consult service rounds to examine a brain injury patient. The patient exhibits open eyes. He follows no commands. He does appear to maintain sleep-wake cycles on EEG. When you pinch his arm, he flexes his elbow. How would you define this patient's arousal status?

vegetative In short, coma is defined as closed eyes, no sleep/wake cycles on EEG, and no purposeful behavior or comprehension. In vegetative state, the patient does have sleep/wake cycles on EEG, and eyes are open, but there is no purposeful activity, only reflexive actions. Minimally conscious state is defined as open eyes, presence of sleep-wake cycles on EEG, and inconsistent awareness of the environment with purposeful behaviors. When this becomes consistent, the patient is said to have "emerged" and is considered to exhibit "normal" arousal at that point, or to at least not possess a disorder of consciousness (DOC).

The most common bone for an osteoporotic fracture in women over age 65 is the:

vertebral

A 28-year-old female runner presents to you with increasing right foot pain. She recently read an article about barefoot running and so has transitioned to that over the last month. She runs six days a week, but more recently she has had to stop running due to pain and has pain throughout the day, worsened by any weight bearing. On exam, there is no swelling or redness. She has tenderness directly over the 3rd metatarsal. No tenderness elsewhere within the foot. Normal neurological exam. Pain with attempts to hop. For your initial management, you decide to:

walking boot for 3 weeks Her history indicates that she has undergone an abrupt change in her training (barefoot running) that would cause a significant change in the types of forces experienced by her skeletal system, and has an accompanying point of focal bony tenderness. Given this history and exam, the patient likely has a 3rd metatarsal stress fracture. This is not considered a "critical" stress fracture site and can be treated initially with a walking boot. If she is able to walk without limping in 3 weeks, she can be progressed out of the boot and to physical therapy. Non-weight bearing in a cast for 6 weeks is generally not needed with "non-critical" stress fractures and would be overly restrictive. Radiographs are relatively insensitive for stress fractures, and would not likely change management in this case as there is little concern for a "critical" stress fracture. Given her likely injury, pain, and functional limitations, returning her to activities at this time would be inappropriate.

How should a PM&R physician define ephaptic transmission?

way of firing a motor unit that has undergone a denervation-reinnervation-denervation cycle A CRD (complex repetitive discharge) is an involuntary discharge that is very wide and serrated (complex in appearance, and repetitive in firing) and occurs due to a motor unit being denervated and then reinnervated by another motor neuron, which itself then becomes denervated. Ephaptic transmission is the process by which these muscle fibers all fire regularly together. CRDs are seen in chronic radiculopathy, anterior horn cell disease, and some normal patients.

Which stroke syndrome has ipsilateral ptosis, external strabismus, a dilated pupil, and contralateral weakness of the lower face, tongue, arm, and leg?

weber Weber syndrome includes the symptoms of ipsilateral ptosis, external strabismus, a dilated pupil, and contralateral weakness of the lower face, tongue, arm, and leg. The chart below delineates each syndrome in detail

What are the characteristics of cervical facet (zygapophyseal) joint mediated pain?

whiplash injury Facet joint (zygapophyseal) pain is frequent in individuals with chronic cervical pain following whiplash injuries. Localizing neck pain to the facet joints can be difficult because facet joint pain referral patterns can overlap discogenic or myofascial pain patterns. Facet joint pain may be difficult to reproduce with palpation and is more common with cervical extension rather than cervical flexion. The referral pattern is typically to the neck and shoulders.

For patients with acute CPP crystal arthritis involving one or 2 joints, the preferred initial management is?

with joint aspiration followed by intra articular glucocorticoid injection. If more than 2 joints = NSAIDS

In the nonoperative treatment of a proximal humerus fracture, when should shoulder passive range of motion (ROM) exercises begin?

within 1 week

what is the diagnostic test for ankylosing spondylitis? lab? or imaging?

x ray of the inflamed Si joints

List the positive predictors for returning to work after SCI:?

younger age male white greater formal education employment at time of injury being able to drive


Conjuntos de estudio relacionados

Project Management and IT: chapter 11

View Set

algebra 2b - unit 2: out of the woods lesson 6-10

View Set

Chapter 8 Understanding Populations

View Set

financial accounting tophat chp 13

View Set